You are on page 1of 46

PRAYAS - 2022

Test – 04
Polity
Topics covered:
 President,Vice-President and Prime Minister
 Governor & Chief minister
 Supreme court and High court
 Union CoM, Cabinet secretary & State CoM
Chief secretary
 Anti defection Laws, 5th & 6th schedule

e
in

PRAYAS-2022
nl
l.o
ria
e
at
cm
ps
.u
w
w
w
w
w
w
.u
ps
cm
at
er
ia
l.o
nl
in
e
Contact us : info@onlyias.com

OnlyIAS Nothing Else Visit : dpp.onlyias.in


Contact : +91-7007 931 912

Q.1) Consider the following statement of Parliament in the absence of Deputy Speaker of Lok
1. The ordinance making power of the President is Sabha
parallel with the legislative power of parliament Choose incorrect Statement/s from the code given
2. An ordinance can last for a maximum of 6 months below:
from the date of its promulgation. a) 1 and 2 only
3. An ordinance can be retrospective in nature. b) 2 only
Select the correct answer from the code given c) 3 only
below: d) 2 and 3 only
a) 1 and 3 only
b) 1 and 2 only Q.5) Consider the following about Prime Minister
c) 2 only 1. Cabinet Committee on Accommodation is
d) 3 only headed by the Prime Minister of India.
2. The National Security Guard is responsible for
the security of the Prime Minister.
3. The Executive Council of the Nuclear Command
Q.2) Which of the following statement/s is
Authority of India is headed by the Prime Minister.
are incorrect?
Select the correct statements from the code given
1. Oath for president is prescribed in schedule III of
below:
the constitution along with oath for council of
a) 1 only
ministers.
b) 1 and 3 only
2. Impeachment against the president can be
c) 2 only
initiated in Lok sabha as well as Rajya sabha.
d) None of these
3. When a president is impeached the Vice president
completes the remainder term.
Select the answer from code given below:
Q.6) Consider the following statements
a) 2 and 3 only
1. President is completely immune from civil and
b) 1 and 3 only
c) 1 and 2 only criminal proceedings during his term in office.
d) 1, 2 and 3 only 2. He can be removed from office on grounds of
Violation of the Constitution.
Q.3) Which of the following is part of Electoral 3. Nominated members of both houses do not
College for President: participate in the impeachment process as they are
1. Nominated Member of Parliament not part of the electoral college to elect the President.
2. Elected Member of Legislative Assemblies Which of the statement/s given above is /are
3. Elected Member of Parliament incorrect?
4. Nominated Member of Legislative Assemblies
5. Elected Member of Legislative Council a) 1 and 2 only
6. Elected Member of Legislative Assemblies of b) 2 and 3 only
Union Territory c) 1 and 3 only
d) 2 only
e

Select the correct answer from the code given below


in

a) 1,2,3 and 5 only


nl

b) 2,3,5 and 6 only


c) 2,3 and 6 only
l.o

Q.7) Consider the following statements about


d) 2,3,4 and 5 only vacancy in office of President:
ria

1. The Vice President becomes President for the


remainder term when the office of President falls
e

Q.4) Consider the following statements:


at

1. Resolution for removal of Vice President can be vacant.


2. In case the office of vice President is also vacant,
cm

initiated only in Rajya Sabha.


2. The vacancy for the post of Vice President must be the Chief Justice of India takes charge as
ps

filled within 6 months President of India.


3. The Vice President presides over the joint sitting 3. The Chief Justice has certain limited powers
.u

when he acts as President of India.


w

PRAYAS TEST 4 1
w
w
Contact us : info@onlyias.com

OnlyIAS Nothing Else Visit : dpp.onlyias.in


Contact : +91-7007 931 912

Which of the statement/s given above is /are 2. The office of Governor is an independent
correct? Constitutional office but it is subordinate to the
a) 2 and 3 only Central Government.
b) 2 only 3. The oath of the office to the Governor is
c) 1 and 2 only administered by the President of India.
d) 3 only 4. The Governor can only appoint members of the
State Public Service Commission. However he
Q.8) Consider the following statement cannot remove them.
1. The emolument of the Governor is decided by Which of the above statements is /are correct?
Parliament. a) 1 and 2 Only
2. The Constitution does not mention any tenure for b) 3 Only
office of Governor. c) 1 and 4 Only
3. Ground of removal of Governor is clearly d) 4 Only
mentioned in Constitution.
Which of the above statements is /are correct?
a) 1 only Q.12) With reference to the Chief Minister of the
b) 1 and 2 only states which of the following statements is/are
c) 2 only correct?
d) 2 and 3 only 1. The Constitution of India does not require that a
person must prove his majority in the legislative
assembly before he is appointed as the Chief
Minister.
Q.9) Consider the following statements
2. He shall be appointed by the governor and holds
1. Governor cannot remit a death sentence.
office during the pleasure of the governor.
2. Governor enjoys constitutional as well as
3. The salary and allowances of the chief minister
situational discretionary Power.
are determined by the parliament.
3. Indian Constitution adopted American model
Select the correct answer using the codes given
where the governor of the state is appointed by
below:
the centre.
a) 1 and 2 only
Which of the above statements is /are correct?
b) 1 and 3 only
a) 2 only
c) 2 and 3
b) 2 and 3 only
only
c) 1 only
d) 1 and 2 only d) 1, 2 and 3.

Q.10) Consider the following statements:


1. Discretionary Powers exercised by the Governor Q.13) The Draft Constitution provided for the direct
cannot be questioned by the Court. election of the Governor but the Constituent
2. The Governor can return a money bill for the Assembly opted for the appointment of the
e

reconsideration of the state legislature.


in

Governor by the President, why?


Which of the statements given above is/are correct? 1. The mode of direct election is more likely to
nl

a) Only 1 create conflicts between the Governor and the


l.o

b) Only 2 Chief Minister.


c) Both 1 and 2
ria

2. The Governor being only a Constitutional


d) Neither 1 nor 2 (nominal) head, there is no point in making
e

elaborate arrangements for his election and


at

Q.11) Regarding the position of Governor, consider spending huge amounts of money.
cm

the following statements: 3. The constituent assembly followed the American


1. The same person cannot be appointed as the model, where the Governor is appointed by the
ps

Governor for two or more states. President.


.u
w

PRAYAS TEST 4 2
w
w
Contact us : info@onlyias.com

OnlyIAS Nothing Else Visit : dpp.onlyias.in


Contact : +91-7007 931 912

4. The election of the Governor would create d) Neither 1 or 2


separatist tendencies and thus affect the political
stability and unity of the country.
Select the correct answer using the codes given
Q.17) Consider the following statements about
below:
removal of High Court Judge:
a) 1 and 2 only
1. The details of process of impeachment of high
b) 3 only
court judge is given in constitution
c) 1, 2 and 4 only
2. The ground for removal of a judge of High court
d) 1, 2, 3 and 4 can be proved misbehaviour, incapacity or
violation of constitution.
Q.14) What are the qualifications laid down by the Which of the statements given above is/are correct?
Constitution for appointment of the governor? a) 1 only
1. He should not belong to the state to which he is b) 2 only
going to be appointed as the governor. c) Both 1 and 2
2. He should not hold any office of profit under the d) Neither 1 or 2
Union government or any state government or
any local authority or any public authority.
3. He should be a citizen of India.
Q.18) Consider the following statements:
4. He should have completed the age of 35 years.
1. The salary allowance of a high court judge is
Which of the statements given above is/are correct?
decided by parliament.
a) 1 and 4 only
2. The salary of a high court judge is charged on a
b) 2, 3 and 4 only
consolidated fund of India.
c) 3 and 4 only
3. The Constitution does not contain detailed
d) 1, 2, 3 and 4 provisions regarding jurisdiction of the high court.
Which of the statements given above is/are correct?
Q.15) Consider the following statements: a) 1 and 3 only
1. There is no provision in the constitution for b) 1 and 2 only
appointment of distinguished jurists as a judge of c) 1 only
the high court. d) All of the above.
2. The Constitution does not prescribe a minimum
age to be appointed as judge of the High Court.
3. The judges of the high court are appointed as well
as administered oath by the governor of state. Q.19) Consider the following statements:
Which of the statements given above is/are correct? 1. Supervisory jurisdiction of the high court extends
a) 1 only to all courts and tribunals in its territorial
b) 2 and 3 only jurisdiction except military court or tribunal.
c) 1 and 2 only 2. Supervisory jurisdiction also covers administrative
d) 1,2 and 3 superintendence.
3. The High court can issue writs to enforce rights
e

within its territorial jurisdiction only.


in

Which of the statements given above is/are correct?


nl

Q.16) Consider the following statements: a) 1 only


l.o

1. The Parliament can increase the number of Judges b) 1 and 2 only


of the Supreme Court and High courts.
ria

c) 2 only
2. The speaker must admit a motion to remove a d) 1 and 3 only
e

judge of HC of any state as per Judges Enquiry Act,


at

1968.
cm

Which of the statements given above is/are correct?


a) 1 only Q.20) With reference to the Supreme Court of India,
ps

b) 2 only consider the following statements-


c) Both 1 and 2 1. Tenure for the judge of the Supreme Court is fixed
.u

by the constitution itself.


w

PRAYAS TEST 4 3
w
w
Contact us : info@onlyias.com

OnlyIAS Nothing Else Visit : dpp.onlyias.in


Contact : +91-7007 931 912

2. The President can issue removal orders for a judge c) 3 only


after the recommendation of the Collegium. d) 1,2 and 3
3. Since the commencement of the constitution no
Judge of the Supreme Court has been impeached. Q.24) With reference to the state council of ministers
Which of the statements given above is/are correct? which of the following statements is/are incorrect?
a) 1 and 2 only 1. The Chief Minister may be a member of any of the
b) 2 and 3 only two houses of the state legislature.
c) 3 only 2. A person who is not a member of the state
d) 1 only legislature can be appointed as Chief Minister for
three months only.
Q.21) With reference to the Judicial Review powers 3. The Chief Minister's resignation or death,
of the Supreme court of India, consider the following automatically dissolves the council of ministers.
statements- 4. Chief Minister is the Vice-chairman of the state
1. The Supreme Court of India has the power to planning board.
review its own judgement. Select the answer using the codes given below:
2. The term Judicial Review is explicitly mentioned a) 1 and 3 only
in the Constitution. b) 2 and 4 only
3. The scope of judicial review evolved to protect c) 2,3 and 4 only
the constitutionally guaranteed fundamental d) 1,2 3 and 4
rights of citizens.
Which of the statements given above is/are correct? Q.25) With reference to the Chief Secretary which of
a) 1 and 2 only the following statements is/are correct?
b) 2 and 3 only 1. The Chief Secretary acts as conscience-keeper of
c) 1 only all state civil servants.
d) 1 and 3 only 2. The chief secretary acts as the residual legatee.
3. The chief secretary plays a supervisory role and
Q.22) Which of the following statement/s is/are acts as a resource person.
correct? Select the correct answer using the codes given
1. The term of Council of Ministers is 5 years. below:
2. The Council of Ministers is headed by the Head of a) 1 and 2 only
Republic of India. b) 2 and 3 only
Select the correct answer from the code given below: c) 1 and 3 only
a) 1 only d) 1,2, and 3.
b) 2 only
c) Both 1 and 2 Q.26) With reference to the functions of the Cabinet
d) Neither 1 or 2 Secretariat which of the following statements is/are
correct?
Q.23) Which of the following statement/s is/are 1. Facilitating smooth transaction of business in
correct? Ministries/ Departments of the Government by
e

assisting in decision-making in Government by


in

1. The Council of Ministers can advise the President


to dissolve Lok Sabha and call for fresh elections ensuring Inter-Ministerial coordination, ironing
nl

at any time. out differences and evolving consensus amongst


l.o

2. The advice rendered by Council of Minister to Ministries/ Departments.


2. It provides Secretarial assistance to the Cabinet
ria

President can be inquired in court of law


3. The total number of ministers, excluding the and its Committees.
e

Prime Minister, in the Council of Ministers shall 3. It helps in Monitoring, Coordination and
at

not exceed 15% of the total strength of the Lok Promoting new policy initiatives
cm

Sabha 4. He plays a supervisory role and as a resource


Select the correct answer from the code given below: person in tendering advice for final decisions to
ps

a) 1 and 2 only the state government.


.u

b) 1 only
w

PRAYAS TEST 4 4
w
w
Contact us : info@onlyias.com

OnlyIAS Nothing Else Visit : dpp.onlyias.in


Contact : +91-7007 931 912

Select the correct answer using the codes given b) 2 only


below: c) Both 1 and 2
a) 1 and 2 only d) Neither 1 nor 2
b) 2 and 3 only
c) 1,2 and 3 only
d) 1,2,3 and 4. Q.30) With reference to the disqualification of
members on grounds of anti-defection which of the
following statements is/are incorrect?
Q.27) Consider the following statements with respect 1. A Nominated member of a House becomes
to the Union Council of Ministers- disqualified to remain a member of the House if
1. Functions of the council of ministers are he joins any political party immediately after such
determined by the Cabinet. election.
2. The Council of Ministers is collectively responsible 2. An Independent member of a House becomes
to the Parliament. disqualified for being a member of the House if he
3. Council of ministers doesn't meet as a body to joins any political party after the expiry of six
transact the government business. months from the date on which he takes his seat
Which of the statements given above is / are in the House.
correct? Select the answer using the codes given below:
a) 1 and 2 only a) 1 only
b) 2 and 3 only b) 2 only
c) 1 and 3 only c) Both 1 and 2
d) 2 only d) Neither 1 nor 2

Q.28) Consider the following statements with respect Q.31) With reference to the exceptions of anti-
to the Union Council of Ministers- defection which of the following statements is/are
1. Every Minister is entitled to vote and speak in correct?
proceedings of either Houses of the Parliament. 1. Anti- Defection law does not apply If a merger
2. The Ministers shall hold office during the takes place when one-third of the members of the
pleasure of the President. party have agreed to such merger.
3. The advice tendered by Ministers to the 2. If a member, after being elected as the presiding
President can be inquired into by any court. officer of the House, voluntarily gives up the
Which of the statements given above is / are membership of his party or re-joins it after he/she
correct? ceases to hold that office.
a) 1 and 2 only Select the correct answer using the codes given
b) 2 only below:
c) 1 and 3 only a) 1 only
d) 2 and 3 only b) 2 only
c) Both 1 and 2
Q.29) With reference to the disqualification of d) Neither 1 nor 2
e

Members of Political Parties on grounds of anti-


in

defection which of the following statements is/are


nl

incorrect? Q.32) With reference to the anti-defection which of


l.o

1. If he voluntarily gives up his membership of such a the following statements is/are incorrect?
political party.
ria

1. The Presiding Officer decision is final and cannot


2. If he votes or abstains from voting in such a house be questioned in any court.
e

contrary to the direction issued by his political 2. The Presiding Officer is empowered to make rules
at

party, with or without obtaining permission and to give effect to the provisions of the tenth
cm

such act has been condoned by the party within schedule.


15 days. 3. The Presiding officer can take up an anti-defection
ps

Select the answer using the codes given below: case on a su-moto basis.
a) 1 only
.u

Select the answer using the codes given below:


w

PRAYAS TEST 4 5
w
w
Contact us : info@onlyias.com

OnlyIAS Nothing Else Visit : dpp.onlyias.in


Contact : +91-7007 931 912

a) 1 only Select the correct answer using the code given


b) 2 only below:
c) 1 and 3 only a) 1, 2 and 3 only
d) 1,2 and 3. b) 1, 3 and 4 only
c) 1, 2 and 4 only
d) 3 and 4 only
Q.33) With reference to the anti-defection law,
which of the following statements is incorrect? Q.36) Which of the following Veto Power is exercised
a) It was added to the Constitution by the 52nd by the President of India?
amendment. 1. Absolute Veto
b) An independent member of a house does not get 2. Qualified Veto
disqualified if he joins a political party within six 3. Suspensive Veto
months from the date on which he takes a seat in 4. Pocket Veto
the house. Select the correct Ans: from the code given below:
c) The law has been amended only once since its a) 1,2 and 3 only
inception. b) 2,3 and 4 only
d) Under the anti-defection law, voluntary c) 1,3 and 4 only
resignation from a political party can lead to d) All of the above
disqualification from the legislature.
Q.37) With reference to the President, consider the
Q.34) How has Anti-defection law impacted the following statements:
dynamics of the Indian Political system ? 1. He decides on disqualification of Members of
1. Has provided for greater stability in the respective Parliament in consultation with the Speaker.
legislatures by checking the propensity of 2. He can seek advice from the Supreme Court and
legislators to change parties. such advice tendered is binding on the President.
2. Has completely rooted out party hopping from 3. The President of India does not possess veto
Indian politics. power with respect to the Constitution
3. Has encouraged the legislators’ right to dissent Amendment Bill.
and freedom of conscience. Which of the statements given above is/are correct?
4. Has given clear-cut constitutional recognition to a) Only 1
the existence of political parties. b) 1 and 2
Select the correct answer using the codes given c) 2 and 3
below: d) Only 3
a) 1, 2 and 3 only
b) 2 and 3 only Q.38) With reference to the President of India,
c) 1, 3 and 4 only Consider the following statements.
d) 1 and 4 only 1. The resolution for impeachment charges of the
president should be signed by one-third member
Q.35) The constitution bars disqualification of a of the house in which charges have been framed.
e

member on the grounds of defection in which of the 2. The Constitution defines the meaning of ground
in

following cases? for the impeachment of the President.


nl

1. Merger of two political parties with one-thirds of Which of the statements given above is/are correct?
l.o

the members of the legislature party agreeing to a) 1 only


such a defection.
ria

b) 2 only
2. Split in a political party with two-thirds members c) Both 1 and 2
e

of the Legislature party agreeing for the split. d) Neither 1 nor 2


at

3. Voluntary resignation from the party by the


cm

presiding officer of a house. Q.39) Consider the following statement about the
4. A nominated member’s decision to join a political vacancy in the president office after expiration of the
ps

party before the expiry of 6 months from the date term of the sitting President:
on which he takes a seat in the house.
.u
w

PRAYAS TEST 4 6
w
w
Contact us : info@onlyias.com

OnlyIAS Nothing Else Visit : dpp.onlyias.in


Contact : +91-7007 931 912

1. An election to fill the vacancy must be completed d) 1 and 4 only


before the expiration of the term of the sitting
President. Q.42) The electoral college of Vice-President is
2. President must continue to hold the office until different from the electoral college of President in
his successor enters upon his office. the following ways.
3. The Vice President acts as the President after the 1. It consists of both elected and nominated
expiration of the term of sitting the President until members of the Parliament of India.
the new President is elected. 2. It includes the elected members of the state
Which of the statements given above is/are correct? legislative assemblies.
a) 1 only 3. It also includes the elected members of the
b) 1, 2 and 3 legislative assemblies of the Union Territories of
c) 1 and 2 only Delhi and Puducherry.
d) 1 and 3 only Which of the statements given above is/are correct?
a) 1 and 2 only
Q.40) With reference to the Ordinance making b) 1 only
power of the President, Consider the following c) 1,2, and 3 only
statements. d) None of the above
1. The power of the President to issue an ordinance
is parallel to the power of legislation. Q.43) With reference to the Prime Minister, consider
2. The President can issue an ordinance only when the following statement.
both the Houses of the Parliament are not in 1. The Constitution of India does not contain any
session. specific procedure for the selection and
3. The President can make an ordinance only when appointment of the Prime Minister.
he is satisfied that the circumstances exist that 2. The term of the office of the Prime Minister is
render it necessary for him to take immediate fixed for 5 years.
action. 3. He/ She can bring about the collapse of the
4. The decision of the President to issue an council of ministers by resigning from office.
ordinance cannot be challenged in a court. 4. He/ She does not have the power to recommend
Which of the statements given above is/are correct? the dissolution of the Lok Sabha to the President.
a) 1 and 3 only Which of the statements given above is/are correct?
b) 2, 3 and 4 only a) 1 only
c) 3 only b) 1 and 2 only
d) 4 only c) 1, 2, 3 and 4
d) 1 and 3 only
Q.41) Consider the following statements.
1. Any of the doubts and disputes relating to the Q.44) Consider the following statements with
election of Vice-President is decided by the reference to power of the Prime Minister in relation
Supreme Court whose decision is final. to Parliament.
2. Election of the Vice president can be challenged 1. Prime minister advises the President for
e

on the ground of any vacancy in the Electoral summoning and proroguing of sessions of the
in

College. Parliament.
nl

3. The Vice-President of India is modeled on the 2. Prime minister announces policies of the
l.o

lines of the Vice-President of America but with government on the floor of the House.
3. Prime minister advises the President with regard
ria

some difference.
4. The Vice-President of India does not assume the to the appointment of important officials like
e

office of the President when it falls vacant for the attorney general of India, CAG, chairman and
at

unexpired term like Vice-President of America. members of UPSC, and so on.


cm

Which of the statements given above is/are correct? 4. Prime minister is the principal channel of
a) 1 and 2 only communication between the President and the
ps

b) 1 and 3 only council of ministers.


Which of the statements given above is/are correct?
.u

c) 1, 3, and 4 only
w

PRAYAS TEST 4 7
w
w
Contact us : info@onlyias.com

OnlyIAS Nothing Else Visit : dpp.onlyias.in


Contact : +91-7007 931 912

a) 1 and 3 only
b) 1 and 2 only Q.48) Consider the following statements.
c) 1, 2 and 3 only 1. The chief justice of the concerned state high court
d) 1, 2, 3 and 4 may be appointed temporarily to discharge the
functions of the governor of that state.
Q.45) With reference to the pardoning power of the 2. The governor can be removed from office on the
President, consider the following statements: ground of the ‘violation of the Constitution’ as
1. The objective of the pardoning power of the stated in the Constitution.
President is to correct any judicial error in the 3. Pleasure of the President for removal of the
operation of the law. governor is justifiable.
2. The pardoning power of the President is similar to Which of the statements given above is/are correct?
the pardoning power of the Governor. a) 1 only
3. It is also used to afford relief from a sentence, b) 2 only
which the President regards as unduly harsh. c) 1 and 3 only
4. The pardoning power is exercised by the President d) 1, 2, and 3
on the advice of the council of ministers.
Which of the statements given above is/are correct? Q.49) With reference to the Governor, consider the
a) 1, 2, and 3 only following statements.
b) 1 and 4 only 1. He/She can grant pardons, reprieves, respites, and
c) 1 and 3 only remission of punishment or suspend, remit and
d) 1,3 and 4 only commute the sentence of any person convicted of
any offence, outside the jurisdiction of the state
Q.46) Consider the following statements : executive power.
1. Discretionary Powers exercised by the Governor 2. The Governor is consulted by the President while
cannot be questioned by the Court. appointing the judges of the concerned high
2. The Governor can return a money bill for the court.
reconsideration of the state legislature. 3. Governor can also pardon the death sentence of a
Which of the statements given above is/are correct? person.
a) Only 1 4. Governor makes appointments, postings and
b) Only 2 promotions of the district judges in consultation
c) Both 1 and 2 with the state high court.
d) Neither 1 nor 2 Which of the statements given above is/are correct?
a) 1 and 2 only
b) 2 and 3 only
Q.47) Regarding the position of Governor, consider c) 3 and 4 only
the following statements: d) 2 and 4 only
1. The same person cannot be appointed as the
Governor for two or more states. Q.50) Consider the following statements.
2. The office of Governor is an independent 1. The discretionary power of the Governor is
e

greater than the discretionary power of the


in

Constitutional office but it is subordinate to the


President.
nl

Central Government.
3. The oath of the office to the Governor is 2. The Governor has only situational discretionary
l.o

administered by the President of India. power and not Constitutional discretion.


ria

4. The Governor can only appoint members of the Which of the statements given above is/are correct?
State Public Service Commission. However he a) 1 only
e

b) 2 only
at

cannot remove them.


Which of the above statements is /are correct? c) Both 1 and 2
cm

a) 1 and 2 d) Neither 1 nor 2


b) 3 only
ps

c) 1 and 4 Q.51) What special powers are given to the Governor


.u

d) 4 only in case of tribal areas under sixth schedule?


w

PRAYAS TEST 4 8
w
w
Contact us : info@onlyias.com

OnlyIAS Nothing Else Visit : dpp.onlyias.in


Contact : +91-7007 931 912

1. The Governor can organize and reorganize the Q.54) Consider the following with reference to the
autonomous districts. powers of the Governor:
2. In case of different tribes in an autonomous 1. He can appoint any member of the state
district, the governor can divide the district into legislative assembly to preside over proceedings
several autonomous regions. when the offices of both the Speaker and the
3. He/ She can declare an area to be a scheduled Deputy Speaker fall vacant.
area. 2. He decides on the question of disqualification of
4. The Governor is empowered to appoint a members of the state legislature in consultation
commission to examine and report on any matter with the Election Commission.
relating to the administration of the autonomous Which of the statements given above is/are correct?
districts or regions. a) 1 only
Which of the statements given above is /are correct? b) 2 only
a) 1, 2 and 3 only c) Both 1 and 2
b) 1, 2 and 4 only d) Neither 1 nor 2
c) 2, 3 and 4 only
d) 1, 2, 3 and 4
Q.55) With reference to the Supreme Court of India,
Q.52) What powers of the Chief Minister are in consider the following statements-
relation to the Council of ministers? 1. The Supreme Court can punish any person for its
1. He allocates and reshuffles the portfolios among contempt.
the ministers. 2. The parliament is not authorised to expand the
2. He brings about the collapse of the council of jurisdiction of the Supreme Court.
ministers by resigning from office. 3. The constitution authorises the president to seek
3. He communicates to the governor of the state all the opinion of the Supreme Court in any matter.
the decisions of the council of the ministers. Which of the above statements is / are incorrect?
4. He announces the government policies on the a) 1 and 2 only
floor of the house. b) 2 and 3 only
Which of the statements given above is/are correct? c) 1 only
a) 1 and 2 only d) All of the above
b) 1, 2, and 3 only
c) 1 and 4 only
Q.56) Consider the following statements regarding
d) 1, 2, 3 and 4
the writ jurisdiction of the supreme court-
1. The jurisdiction of the Supreme Court is exclusive
Q.53) Consider the following statements:
regarding writs.
1. There shall be a council of ministers with the Chief
2. An aggrieved person has to compulsorily
Minister as the head to aid and advise the
approach the High Court first for the enforcement
Governor for the exercise of his functions unless
of their fundamental rights.
he wants to exercise his functions on his
3. The writ jurisdiction of the high court is wider
discretion.
than that of the Supreme Court.
e

2. It shall be the duty of the Council of Ministers to


in

Which of the statements given above is/are correct?


communicate to the Governor of the state all
a) 1 and 3 only
nl

decisions of the Council of Ministers.


b) 2 and 3 only
l.o

3. The Council of Ministers shall be collectively


c) 2 only
responsible to the Legislative council of the State
ria

d) 3 only
Which of the statements given above is/ are correct?
e

a) 1 and 3 only
at

b) 1 only Q.57) Consider the following pairs -


cm

c) 2 and 3 only
d) 1, 2, and 3 Writs Implications
1. Certiorari Issued on Grounds of
ps

excess of jurisdiction.
.u
w

PRAYAS TEST 4 9
w
w
Contact us : info@onlyias.com

OnlyIAS Nothing Else Visit : dpp.onlyias.in


Contact : +91-7007 931 912

2. Mandamus Prevents illegal holding of Which of the statements given above is/are correct?
public office by a person. a) 1 and 3 only
3. Habeas corpus Protection of an b) 1 and 2 only
individual against arbitrary detention. c) 2 and 3 only
Which of the above pairs given above is / are d) All of the above
correct?
a) 1 and 3 only Q.61) Which of the following constitutes the original
b) 2 and 3 only jurisdiction of High Court-
c) 3 only 1. Enforcement of fundamental rights of citizens.
d) 2 only 2. Disputes relating to election of members of
parliament.
Q.58) In context of high court's, consider the 3. Matters of contempt of court
following statements- Select the correct Ans: using the code below-
1. The judges of the High Court are appointed by the a) 1 and 2 only
governor of the respective state. b) 3 and 4 only
2. The constitution has prescribed a minimum age c) 1 only
for appointment as judge of the High Court. d) All of the above
3. A judge of a High Court can be removed from his
office by the president only. Q.62) Consider the following statements with respect
Which of the following statements given above is / to the Supreme Court of India:
are correct? 1. The constitution of India provides for Collegium
a) 1 and 2 only system for appointment of judges
b) 2 and 3 only 2. The Chief Justice of India can appoint a judge of a
c) 3 only High Court as an ad hoc judge of the Supreme
d) 2 only Court.
3. No Judge can take up the matter on his own,
Q.59) With reference to the high court of India, unless allocated by the Chief Justice of India.
consider the following statements- Which of the statements given above is / are
1. A High Court is barred from considering the correct?
constitutional validity of any Central law. a) 1 and 2 only
2. The law of a high court is binding on all b) 2 and 3 only
subordinate courts functioning within its c) 2 only
territorial jurisdiction. d) 1 and 3 only
3. A High Court has the power to review and correct
its own judgement. Q.63) Consider the following statements regarding
Which of the statements given above is / are High Courts
correct? 1. It has power of superintendence over all courts
a) 1 and 2 only and tribunals in its territorial jurisdiction including
b) 2 and 3 only military courts
e

c) 1 and 3 only 2. It has the power to review and correct its own
in

d) All of the above. judgment as the specific power of review is


nl

conferred to it by Constitution
l.o

Q.60) Consider the following statements with respect 3. It has the power of judicial review to examine the
to the High courts of India: constitutionality of orders and laws of both the
ria

1. The Parliament can extend the jurisdiction of the Central and State governments
e

High Court to any union territory. Which of the statements given above is/are correct?
at

2. the salaries, allowances and pension of a High a) 1 and 2


cm

Court judge is charged on the consolidated fund b) 2 and 3


of the state. c) 3 only
ps

3. The Writ jurisdiction of the high court falls under d) 1, 2 and 3


the basic structure of the constitution.
.u
w

PRAYAS TEST 4 10
w
w
Contact us : info@onlyias.com

OnlyIAS Nothing Else Visit : dpp.onlyias.in


Contact : +91-7007 931 912

Q.64) Consider the following statements d) 2 and 3


1. The salaries, allowances, privileges of the judges
of the High Court are determined by the State Q.68) Consider the following statements
legislature 1. The appellate jurisdiction of a High Court is wider
2. The Constitution does not contain detailed than its Original Jurisdiction
provisions with regard to the jurisdiction and 2. The provisions of Articles 13 and 226 explicitly
powers of a High Court confer the powers of Judicial Review on a High
Which of the statements given above is/are correct? Court
a) 1 only Which of the statements given above is/are correct?
b) 2 only a) 1 only
c) Both 1 and 2 b) 2 only
d) Neither 1 and 2 c) Both 1 and 2
d) Neither 1 and 2
Q.65) Consider the following statements regarding
qualification for the Appointment of District Judge Q.69) Which of the following provisions were added
1. He/she should not already be in the service of to the Constitution by the 91st Amendment Act?
Central or State government 1. The total number of ministers, including the Prime
2. He/she should have been an advocate or a Minister, in the Central Council of Ministers shall
pleader for 10 years not exceed 15 per cent of the total strength of the
3. He/she should be recommended by the Supreme parliament.
Court for appointment 2. A Member of Parliament disqualified on the
Which of the statements given above is/are correct? ground of defection shall be disqualified to be
a) 1 only appointed as a Minister.
b) 1 and 2 only 3. A Member of Parliament disqualified on the
c) 2 and 3 only ground of defection shall be disqualified to hold
d) 1 and 3 only any remunerative political post.
Select the correct Ans: using the code given below:
Q.66) Consider the following statements regarding a) 1 and 2 only
the Transfer of Judges b) 2 and 3 only
1. The President can transfer a judge from one High c) 1 and 3 only
court to another after consulting the Governor of d) All of the Above
the state
2. Only the judge who is transferred can challenge Q.70) With reference to the anti-defection law,
the arbitrariness of transfer. consider the following statements:
Which of the statements given above is/are correct? 1. The constitution empowers the President to make
a) 1 only rules to give effect to the provisions of the tenth
b) 2 only schedule.
c) Both 1 and 2 2. According to the rules, the presiding officer is
d) Neither 1 and 2 empowered to take up a defection case suo motu.
e

3. Any question regarding the disqualification of a


in

Q.67) Consider the following statements member arising out of defection is to be decided
nl

1. President can establish a common High Court for by the Presiding officer of the house.
l.o

two or more states Which of the above statements is incorrect?


2. The parliament can extend the jurisdiction of a a) 1 and 2 only
ria

High Court to any Union Territory b) 2 and 3 only


e

3. A person appointed as a judge of High court has to c) 1 and 3 only


at

make and subscribe oath before the Governor d) All of the above
cm

Which of the statements given above is/are correct?


a) 1 and 2 Q.71) In the context of the Cabinet Secretary of
ps

b) 1 and 3 India, consider the following statements-


c) 2 only
.u
w

PRAYAS TEST 4 11
w
w
Contact us : info@onlyias.com

OnlyIAS Nothing Else Visit : dpp.onlyias.in


Contact : +91-7007 931 912

1. Cabinet Secretary is the senior-most civil servant Select the correct Ans: using the codes below-
in India. a) 1 only
2. She/He ranks on the same level as the Attorney b) 1 and 2 only
General of India in the order of precedence. c) 1 and 3 only
3. A cabinet Secretary is appointed for a fixed tenure d) 2 and 3 only
of two years.
Which of the statements given above is/are correct? Q.75) Consider the following statements about 5th
a) 1 and 2 only schedule of Indian Constitution
b) 2 and 3 only 1. President is empowered to declare an area as
c) and 3 only scheduled area
d) All of the above 2. Governor can alter the boundary of Scheduled
areas.
Q.72) Consider the following statements regarding 3. Tribes advisory councils can only be established in
the chief secretary of state in India- states which have areas under 5th schedule.
1. Chief-Secretary acts as a secretary to the Central Which of the statements given above is/are correct?
cabinet of India. a) 1 and 2 only
2. He looks after all those matters which do not fall b) 1 only
within the purview of other secretaries. c) 2 only
3. The civil services board is headed by the Chief d) 1 and 3 only
Secretary of a state.
Which of the statements given above is / are
incorrect?
Q.76) Consider the following statements about Sixth
a) 1 only
schedule
b) 2 and 3 only
1. Sixth schedule extends to state of
c) 3 only
Assam,Meghalaya, Mizoram and Manipur
d) 1 and 3 only
2. The tribal area of the 6th schedule falls outside
the executive authority of the concerned state.
Q.73) Consider the following statements regarding
3. Governor can reorganize and alter the boundaries
the Union Council of Ministers-
of autonomous districts under the schedule.
1. The Council of Ministers Immediately ceased to
Which of the statements given above is/are
hold office immediately after the dissolution of
incorrect?
the Lok Sabha.
a) 1 and 3 only
2. Council of Ministers exercise control over the
b) 1 and 2 only
appointments of the constitutional authorities.
c) 2 and 3 only
3. The president can ask the Council of Ministers to
d) None of the above
reconsider the advice tendered by them.
Which of the statements given above is/are correct?
a) 2 and 3 only
b) 1 only Q.77) Consider the following statements about
e

c) 1 and 3 only autonomous districts under 6th schedule:


in

d) 3 only 1. The elected and nominated members of


nl

autonomous districts hold term for 5 years


l.o

Q.74) Consider the following Pairs- 2. The district Council is empowered to collect
Concept revenue in autonomous districts under 6th
ria

Meaning schedule.
e

1. Collective Responsibility Cabinet 3. The laws made by district council administrators


at

decisions binding on all cabinet ministers require assent of the Governor.


cm

2. Parliamentary Responsibility Duty of the Which of the statements given above is/are correct?
Ministers to hold the Parliament sessions diligently. a) 1 and 2 only
ps

3. Individual Responsibility Ministers hold b) 2 and 3 only


office during the pleasure of the Governor. c) 1 and 3 only
.u
w

PRAYAS TEST 4 12
w
w
Contact us : info@onlyias.com

OnlyIAS Nothing Else Visit : dpp.onlyias.in


Contact : +91-7007 931 912

d) 1,2 and 3 c) Chief Minister of the concerned state


d) Union Minister of Tribal Affairs

Q.78) Which one of the following Schedules to the Q.82) Consider the following statements:
Constitution of India provides for setting up of 1. An amendment to the fifth and sixth schedules of
Autonomous District Councils? the constitution would require the consent of the
a) Third schedule state legislatures.
b) Fourth schedule 2. The constitution requires the President to appoint
c) Fifth schedule a Commission to report on the administration of
d) Sixth schedule the scheduled areas.
3. The acts of Parliament or the State Legislature
Q.79) The provisions of the fifth schedule of the either do not apply to autonomous districts and
Constitution of India shall not apply to which of the autonomous regions or apply with specified
following states? modifications and exceptions.
1. Jharkhand and Arunachal Pradesh Which of the above statements is/are correct?
2. Tripura and Meghalaya a) 1 and 2 only
3. Assam and Mizoram b) 2 only
4. Orissa and Manipur c) 1 and 3 only
Select the correct Ans: using the code given below: d) 2 and 3 only
a) 1 and 4 only
b) 2 and 3 only Q.83) Consider the following statements.
c) 2 and 4 only 1. The presiding officer can take up a defection case
d) 1 and 3 only suo moto.
2. The presiding officer of a House is empowered to
Q.80) With reference to Fifth Schedule of the make rules to give effect to the provisions of the
constitution of India, consider the following Tenth Schedule of the Constitution.
statements: Which of the above statement is/are correct?
1. The President of India has the power to declare a a. 1 only
particular region as a scheduled area. b. 2 only
2. When required by the President, the Governor c. Both 1 and 2
has to submit a report regarding the d. Neither 1 nor 2
administration of such areas.
3. Each state with a scheduled area may or may not
establish a Tribes Advisory Council to advise on
Q.84) As per the Anti-defection provisions a member
the welfare and advancement of the scheduled
of either House of Parliament or either House of a
tribes.
State Legislature belonging to any political party who
Which of the above statements is/are correct?
is disqualified on the ground of defection shall also
a) 1 and 2 only
be disqualified to
b) 2 and 3 only
e

1. Vote or contest in elections


c) 1 and 3 only
in

2. Be appointed as a Minister
d) All of the above
nl

3. Hold any remunerative political post


Select the correct Ans: using the codes below.
l.o

Q.81) “The office of this political entity is


a) 3 only
empowered to direct that any particular act of
ria

b) 2 and 3 only
Parliament or the state legislature does not apply to
c) 1, 2 and 3
e

a scheduled area. It can also make regulations for the


at

d) 1 and 3 only
peace and good government of a scheduled area
cm

after consulting the Tribes Advisory Council.”


The entity referred to in the above statement is: Q.85) Consider the following statements
ps

a) Governor of the concerned state 1. The Ministers hold office during the pleasure of
b) President of India President
.u
w

PRAYAS TEST 4 13
w
w
Contact us : info@onlyias.com

OnlyIAS Nothing Else Visit : dpp.onlyias.in


Contact : +91-7007 931 912

2. The advice tendered by Ministers to the President c) 2 only


can be inquired into any court d) 1, 2 and 3
3. The salaries, allowances of ministers shall be
determined by the President Q.89) Consider the following statements regarding
Which of the statement/s given above is/are Parliamentary Secretaries
correct? 1. They often hold the rank of minister of state and
a) 1 only are appointed by chief minister of the state
b) 1 and 2 only concerned.
c) 1 and 3 only 2. They have no department in their control and are
d) 2 and 3 only attached to senior ministers.
Which of the statements given above is/are
Q.86) Consider the following statements incorrect?
1. There is provision in Indian Constitution for the a) 1 only
legal responsibility of a minister b) 2 only
2. Even after the dissolution of the Lok Sabha , the c) Both 1 and 2
council of ministers does not cease to hold office d) Neither 1 and 2
Which of the statement/s given above is/are
correct? Q.90) Consider the following statements regarding
a) 1 only Cabinet Secretariat.
b) 2 only 1. It assists in decision making in government by
c) Both 1 and 2 ensuring inter-ministerial coordination and
d) Neither 1 and 2 evolving consensus.
2. It is under the direct control of Home Minister.
Q.87) Consider the following statements regarding 3. The administrative head of the Secretariat is the
Cabinet Chief Secretary who is also the ex-officio
1. It is a smaller body and includes the cabinet Chairman of the Civil Services Board.
ministers only Which of the statement/s given above is/are
2. The word cabinet was inserted in Article 352 of correct?
the Constitution in 1978 by the 42nd a) 1 only
Constitutional Amendment Act b) 2 and 3
3. It is an advisory body to the President and its c) 1 and 2
advice is binding on him/her. d) 1 and 3
Which of the statements given above is/are correct?
a. 1 and 2 Q.91) Consider the following statements regarding
b. 2 and 3 Chief Secretary
c. 1 and 3 1. It acts as the head of the state civil services
d. 3 only 2. He/She is the administrative head of the Union
Cabinet secretariat
3. It acts as the Principal advisor to Chief minister on
e

Q.88) Consider the following statements regarding


in

State Council of Ministers all the matters of State Administration


Which of the statements above given is/are correct?
nl

1. Every minister is entitled to vote even if he/she is


not the member of that house. a) 1 and 2
l.o

2. All executive action of the State Government shall b) 1 only


ria

be expressed to be taken in the name of the c) 1 and 3


Governor. d) 1, 2 and 3
e
at

3. The Constitution does not specify exact size of the


State Council of Ministers or the ranking of Q.92) Consider the following statements:
cm

Ministers. 1. Post of Vice president of Indian Constitution has


Which of the statements given above is/are correct?
ps

been taken from Canadian constitution


a) 1 and 3 2. The resolution for removal of the Vice President
.u

b) 2 and 3 can be introduced only in Rajya Sabha.


w

PRAYAS TEST 4 14
w
w
Contact us : info@onlyias.com

OnlyIAS Nothing Else Visit : dpp.onlyias.in


Contact : +91-7007 931 912

3. The resolution for removal of Vice President must a) 1 only


be passed by effective majority in Rajya Sabha and b) 2 only
Lok Sabha c) Both 1 and 2
Which of the statements given above is/are correct? d) Neither 1 nor 2
a) 1 and 3 only
b) 2 only
c) 1 and 2 only Q.96) Consider the following statements
d) 2 and 3 only 1. The constitution has provided for a maximum
number of 10 judges in the High Court.
Q.93) Consider the following statements 2. The jurisdiction of High Court regarding union
1. The office of the governor and the manner of his territory is decided by the President of India
appointment has been taken from Canadian 3. Disputes regarding election of Member of
constitution Parliament falls under its original jurisdiction of
2. He is entitled to such emoluments, allowances the high court.
and privileges as may be determined by Which of the statements given above is/are correct?
Parliament. a) 1 and 3 only
3. When the same person is appointed as the b) 3 only
governor of two or more states, the emoluments c) 2 and 3 only
and allowances payable to him are shared by the d) None of the above.
states in equal proportion
Which of the statements given above is/are
incorrect?
Q.97) Which of the following falls under original
a. 2 only
jurisdiction of high court:
b. 1 and 2 only
1. Matters related to revenue collection.
c. 1 and 3 only
2. Enforcement of fundamental rights of citizens
d. 3 only
3. Cases ordered to be transferred from a
subordinate court involving the interpretation of
the Constitution.
Q.94) Consider the following statements about 4. Disputes relating to election of Panchayat
Governor: members.
1. The Governor has been provided security of Select the correct option/s from the code given
tenure of 5 years in the constitution. below
2. The Governor has the power to appoint and a) 1 and 2 only
remove members of the state information b) 2 and 3 only
commission. c) 1,2 and 3 only
3. The Governor cannot pardon the death sentence d) All of the above.
of a prisoner.
Which of the statements given above is/are correct?
e

a) 1 and 2 only
Q.98) Consider the following statements regarding
in

b) 2 only
anti defection law:
nl

c) 3 only
1. Disqualification on the ground of defection for a
d) 2 and 3 only
l.o

Member of Parliament will not apply if a two-third


member goes out of his party as a result of a
ria

merger of the party with another party.


e

Q.95) Consider the following statements: 2. Presiding officer of the house decides the
at

1. The number of judges of Supreme court and High question of disqualification of a member of the
cm

court is decided by Parliament of India house arising on the ground of defection


2. There is no minimum age prescribed to be 3. The decision of the presiding officer cannot be
ps

appointed as Judge of Supreme Court. questioned in a court of law.


Which of the statements given above is/are correct? Which of the statements given above is/are correct?
.u
w

PRAYAS TEST 4 15
w
w
Contact us : info@onlyias.com

OnlyIAS Nothing Else Visit : dpp.onlyias.in


Contact : +91-7007 931 912

a) 1 only
b) 1 and 2 only
c) 1 and 3 only
d) 2 only

Q.99) Consider the following statements


1. The Salary of the chief minister of a state is
decided by an act of parliament.
2. The term of the chief minister is for 5 years.
Which of the statements given above is/are
incorrect?
a) 1 only
b) 2 only
c) Both 1 and 2
d) None of the above

Q.100) Consider the following statements about


Cabinet Secretary:
1. He is the ex-officio head of the Civil Services Board
and the Cabinet Secretariat.
2. He Provides Secretarial assistance to the Cabinet
and its Committees.
3. He evolves consensus through the instrumentality
of the standing Committees of Secretaries.
Which of the statements given above is/are correct?
a) 1 and 2 only
b) 2 and 3 only
c) 1 and 3 only
d) 1,2 and 3

e
in
nl
l.o
e ria
at
cm
ps
.u
w

PRAYAS TEST 4 16
w
w
Contact us : info@onlyias.com

OnlyIAS Nothing Else Visit : dpp.onlyias.in


Contact : +91-7007 931 912

Q.1) Ans: d Exp:


Exp: Electoral College of President consists of:
· Statement 1 is incorrect: Article 123 of the · Elected Members of Legislative assembly all
Constitution grants the President certain law- states
making powers to promulgate Ordinances · Elected Members of the Legislative
when either of the two Houses of Parliament is Assembly of Union Territories having Legislature
not in session and hence it is not possible to · Elected members of Lok Sabha
enact laws in the Parliament. There is limitation · Elected members of Rajya Sabha.
on the President to promulgate ordinance when The nominated members of the State Legislative
Parliament is not in session. Hence it is not a Assembly, Nominated members of Parliament
parallel power with the legislature. and Members of Legislative Council(Elected as
well as Nominated) are not part of the Electoral
· Statement 2 is incorrect: An ordinance will
College.
expire after 6 weeks once both the houses of
Parliament are in session. Thus, the maximum
Q.4) Ans: d
validity of an ordinance is 6 months and 6 weeks Exp:
· Statement 3 is correct: An ordinance may
have retrospective effect and may be modify · Statement 1 is correct: The Vice-President
repeal any act of Parliament or even another may be removed from his office by a resolution
ordinance of the Rajya Sabha by a majority of all the
Source: members of the upper house and agreed to by
https://www.prsindia.org/tags/ordinances the Lok Sabha. No such resolution shall be moved
unless at least fourteen days’ notice has been
Q.2) Ans: b given of the intention to move the resolution.
Exp: · Statement 2 is incorrect: Article 68(2) of the
Constitution provides that an election to fill a
· Statement 1 is incorrect: The Oath vacancy in the office of Vice-President occurring
of Council of Ministers is there in Schedule III of by any reason shall be held as soon as possible. It
Constitution. Council of Ministers takes oath of is further provided that the person elected to fill
office and secrecy. The oath of President is the vacancy shall be entitled to hold office for
provided in Article 60 of constitution. the full term of five years from the date he
· Statement 2 is correct: The impeachment enters upon his office.
proceedings can be initiated by either house of · Statement 3 is incorrect: The joint sitting of
Parliament. These charges should be signed by the Parliament is called by the President. The
one fourth of members of the house that had hierarchy to preside over the joint sitting of
e

initiated the impeachment proceedings. For Parliament are as follows:


in

Example, in Lok Sabha it should be signed by one-


nl

fourth of 543 members.  The Speaker of Lok sabha


l.o

· Statement 3 is incorrect: When the  The Deputy speaker of lok sabha


ria

President is impeached Vacancy must be filled  The Deputy chairperson of Rajya sabha
within 6 months. The Vice President acts as
e

President till the new President takes oath for the


at

If any of the above officers are not present than


office.
cm

any other person as may be determined by the


Source: Laxmikanth member present at the joint sitting , Presides
ps

over the meeting. Thus, the Vice President does


Q.3) Ans: c not Preside over the joint sitting of Parliament.
.u
w

PRAYAS TEST 4 17
w
w
Contact us : info@onlyias.com

OnlyIAS Nothing Else Visit : dpp.onlyias.in


Contact : +91-7007 931 912

Source: Laxmikanth Constitution’.


· Statement 3 is incorrect: An impeachment is
Q.5) Ans: d a quasi-judicial procedure in the Parliament. In
Exp: this context, two things should be noted:
o the nominated members of either House of
· Statement 1 is incorrect: The Cabinet Parliament can participate in the impeachment
Committee on Accommodation is headed by of the President though they do not participate in
the Home Minister. It is to be noted that cabinet his election
committee on parliamentary affairs is also o the elected members of the legislative
headed by Home Minister assemblies of states and the Union Territories of
· Statement 2 is incorrect: The organization Delhi and Puducherry do not participate in the
responsible for Security of the Prime Minister is impeachment of the President though they
the Special Protection group. It also looks after participate in his election.
the security of Prime Minister Close Kin of Prime
Minister. Q.7) Ans: b
· Statement 3 is incorrect: Exp:

 The Prime Minister Heads the Political Council of · Statement 1 is incorrect: When a vacancy
the Nuclear Command Authority. He is the final occurs in the office of the President due to his
authority to decide upon the use of nuclear resignation, removal, death or otherwise,
weapons. the Vice-President acts as the President until a
 The Executive Council gives its suggestion to new President is elected. Further, when the
Political Council of Nuclear Command Authority, sitting President is unable to discharge his
The Executive Council is headed by National functions due to absence, illness or any other
Security Advisor. cause, the Vice- President discharges his
functions until the President resumes his office.
Source: · Statement 2 is correct: In case the office of
https://theprint.in/india/heres-the-full-list-of- Vice-President is vacant, the Chief Justice of
cabinet-committees-after-reshuffle-pm-heads-6- India (or if his office is also vacant, the senior
amit-shah-2/246277/ most judge of the Supreme Court available) acts
as the President or discharges the functions of
the President.
Q.6) Ans: c · Statement 3 is incorrect: When any person,
Exp: ie, Vice-President, chief justice of India, or the
senior most judge of the Supreme Court is acting
· Statement 1 is incorrect: During his term of as the President or discharging the functions of
e

office, he is immune from any criminal the President, he enjoys all the powers and
in

proceedings, even in respect of his personal acts. immunities of the President and is entitled to
nl

He cannot be arrested or imprisoned. such emoluments, allowances and privileges as


l.o

However, after giving two months’ notice, civil are determined by the Parliament.
ria

proceedings can be instituted against him during


his term of office in respect of his personal acts. Q.8) Ans: a
e
at

· Statement 2 is correct: The President can Exp:


be removed from office by a process of
cm

impeachment for ‘violation of the Constitution’. Statement 1 is correct:


ps

However, the Constitution does not define the


meaning of the phrase ‘violation of the
.u
w

PRAYAS TEST 4 18
w
w
Contact us : info@onlyias.com

OnlyIAS Nothing Else Visit : dpp.onlyias.in


Contact : +91-7007 931 912

 The Governor shall be entitled without payment  While exercising function as administrator
of rent to the use of his official residences and of Union Territory
shall be also entitled to such emoluments,
allowances and privileges as may be determined Some Situational Discretion are as follows:
by the law of parliament.
 When the same person is appointed as Governor  Appointment of Chief Minister when no party has
of two or more States, the emoluments and clear majority
allowances payable to the Governor shall be  Dismissal of Council of Minister when they cannot
allocated among the States in such proportion as prove confidence in house
determined by the president.
Statement 3 is incorrect: The manner of
appointment of Governor has been inspired by
Statement 2 is incorrect: Under Article 156 it is the Canadian Constitution where the Governor
given that the governor is appointed by the is appointed by the Centre (not directly elected).
President of India for a term of five years and To maintain the parliamentary form of
holds office at the President's pleasure. Government at state level the model of the USA
(direct election) was dropped and Canadian
Statement 3 is incorrect: The President has the model was accepted.
power to remove a Governor at any time without Source: Laxmikanth
giving him or her any reason and without granting
an opportunity to be heard. The governor holds Q.10) Ans: d
office as long as the President desires and there is Exp:
no clear ground of his removal mentioned in the
Constitution. Statement 1 is Incorrect: The Discretionary
powers exercised by the Governor is not beyond
Reference Que- 2018 Set D Q.96 Judicial review. The Supreme Court, speaking
Source-Laxmikanth through a five-judge Bench in Nabam Rebia and
https://www.prsindia.org/theprsblog/removal- Bamang Felix v. Deputy Speaker (2016) ruled
governors-what-does-law-say that the discretionary power of the Governor is
extremely limited and entirely amenable to
Q.9) Ans: a judicial review.
Exp: Statement 2 is Incorrect: As the money bill is
introduced with the Governor’s permission, he
Statement 1 is incorrect: The Governor cannot cannot return the money bill for the
pardon a death sentence. Even if state law reconsideration of the State Legislature. He can
prescribes a death sentence, the power to grant either give his assent, withhold his assent or
e

pardon is with the President. However the reserve the bill for consideration of the president.
in

governor can suspend, remit or commute a Source: https://www.thehindu.com/opinion/op-


nl

death sentence. ed/governors-discretion/article25021561.ece


l.o
ria

Statement 2 is correct: Q.11) Ans: d


The Governor enjoys Constitutional discretion in: Exp:
e
at

 Reservation of bill when for Presidential Statement 1 is Incorrect: Usually, there is a


cm

consideration governor for each state, but the 7th


ps

 Recommendation of Presidents rule in a state Constitutional Amendment Act of 1956 facilitated


the appointment of the same person as a
.u
w

PRAYAS TEST 4 19
w
w
Contact us : info@onlyias.com

OnlyIAS Nothing Else Visit : dpp.onlyias.in


Contact : +91-7007 931 912

governor for two or more states. and there could not be two elected executive
Statement 2 is Incorrect: Supreme Court in 1979, heads in the state.
the office of governor of a state is not an · Statement 2 is correct: Being only a
employment under the Central government. It is constitutional head of the state the election of
an independent constitutional office and is not the governor is entirely unnecessary also the
under the control of or subordinate to the system of the presidential nomination enables
Central government the Centre to maintain its control over the states.
Statement 3 is Incorrect: The oath of office to · Statement 3 is incorrect: The Constituent
the Governor is administered by the Chief justice Assembly had not followed the American model,
of the concerned state high court and in his in America the Governor of a state is directly
absence, the senior-most judge of that court elected and not appointed by the President. The
available( not the President) Constituent Assembly followed the Canadian
Statement 4 is Correct: The Governor appoints model, where the Governor of a state is
the chairman and members of the State public appointed by the Governor-General (Centre).
service commission. However, they can be · Statement 4 is correct: If the governor is
removed only by the president and not by a directly elected there will be two elected bodies
governor. which would try to persuade each other and use
their powers against each other which would
Q.12) Ans: a have created separatist tendencies and thus
Exp: affect the political stability and unity of the
country.
Statement 1 is correct: - The Constitution does Source: M. LAXMIKANTH
not require that a person must prove his majority
in the legislative assembly before he is appointed Q.14) Ans: c
as the Chief Minister. The governor may first Exp:
appoint him as the Chief Minister and then ask
him to prove his majority in the legislative · Statement 1 is incorrect: He should be an
assembly within a reasonable period. outsider, that is, he should not belong to the
Statement 2 is correct: Article 164 only says that state where he is appointed, so that he is free
the Chief Minister shall be appointed by the from the local politics. It is a developed
governor. He holds office during the pleasure of convention over the years and not the
the governor. However, this does not mean that qualification specified in the Constitution.
the governor can dismiss him at any time. He · Statement 2 is incorrect: This qualification is
cannot be dismissed by the governor as long as not mentioned in the Constitution for the
he enjoys the majority support in the legislative governor but it is mentioned in the Constitution
assembly for the qualification of the President.
e

Statement 3 is incorrect: The salary and · Statement 3 is correct: A person who wants
in

allowances of the Chief Minister are to be the governor of a state needs to be a citizen
nl

determined by the state legislature. of India.


l.o

· Statement 4 is correct: In order to be


ria

Q.13) Ans: c appointed as a governor of the state, the person


Exp: needs to complete the age of 35 years.
e
at
cm

· Statement 1 is correct: Direct election of the Q.15) Ans: c


governor would create a conflict against the Chief Exp:
ps

Minister, as the Chief Minister is already an


elected representative of the people in the state Statement 1 is correct: The Chief Justice of a High
.u
w

PRAYAS TEST 4 20
w
w
Contact us : info@onlyias.com

OnlyIAS Nothing Else Visit : dpp.onlyias.in


Contact : +91-7007 931 912

Court is appointed by the President with the Statement 2 is incorrect: Constitution provides
consultation of the Chief Justice of the Supreme for ground to remove judges of High court which
Court and the Governor of the State. are proved misbehaviour or incapacity. Violation
Qualifications for the Judges are: of the Constitution is not a ground of removal of
a judge of higher Judiciary.
 He should be a citizen of India.
 He should have been Held judicial office in the Q.18) Ans: a
territory of India for 10 years or an Advocate for Exp:
10 years in a High Court.
Statement 1 is correct: The salary of High court
The Constitution does not prescribe an eminent judges is charged Expenditure which is decided
jurist to become judge of the High Court as is the by parliament of the country.
case with the Supreme Court.
Statement 2 is incorrect: The salary of a high
Statement 2 is correct: As is clear from the above court judge is charged on a consolidated fund of
explanation the Constitution does not prescribe a States.
minimum age to be appointed as judge of the
High Court. Statement 3 is correct: The given statement is
true. Nothing substantial regarding jurisdiction of
Statement 3 is incorrect: Oath to the judges of the high court is provided in the Constitution
the high court is administered by the governor of
state or person appointed by him. However, they Q.19) Ans: b
are appointed by President of India Exp:

Q.16) Ans: d Statement 1 is correct: Supervisory Jurisdiction is


Exp: a special power enjoyed by the High Court.
It extends to all courts that lie within the
Statement 1 is incorrect: The number of Judges of the territorial jurisdiction of the High Court. It is to be
Supreme Court are decided by Parliament. However, noted that it does not extend to military courts.
the number of Judges of high courts are decided by
the President.
Statement 2 is correct: Supervisory Jurisdiction of
Statement 2 is incorrect: Removal proceedings
the High Court extends to not only judicial
against a Supreme Court or a High Court judge can be but also in administrative matters like paying
initiated in any of the houses of Parliament. For this: A wages to court employees.
minimum of 100 members of Lok Sabha or 50
members of Rajya Sabha may give a signed notice to Statement 3 is incorrect: Article 226 of the
e

the speaker/Chairman. The speaker/Chairman may or Constitution empowers a high court to issue writs
in

may not admit the motion of removal. for the enforcement of the rights of the citizens
nl

and for any other purpose. The high court can


l.o

Q.17) Ans: d issue writs not only within its territorial


ria

Exp: jurisdiction but also outside its territorial


jurisdiction if the cause of action arises within its
e
at

Statement 1 is incorrect: The Constitution does territorial jurisdiction.


cm

not contain detailed provision to remove judges


of the High Court. It is followed as per the
ps

provision of Judicial Inquiry Act, 1968.


Q.20) Ans: c
.u
w

PRAYAS TEST 4 21
w
w
Contact us : info@onlyias.com

OnlyIAS Nothing Else Visit : dpp.onlyias.in


Contact : +91-7007 931 912

Exp: · Statement 2 is incorrect- The term Judicial


Review is not explicitly mentioned in the
· Statement 1 is incorrect- The Constitution Constitution. However, Some provisions in the
has not fixed any tenure for the judge of the constitution in the support of the judicial review
Supreme Court. are:
But, it makes the following provisions in this Article 13 declares that any law which
matter- contravenes any of the provisions of the part of
1. He/she holds office until he attains the age Fundamental Rights shall be void.
of 65 years. Any question regarding his age is to Articles 32 and 226 gives the roles of the
be determined by such authority and in such protector and guarantor of fundamental rights to
manner as provided by Parliament. the Supreme Court and High Courts.
2. He/she can resign his office by writing to the · Statement 3 is correct- Judicial review
President. evolved to protect the constitutionally
3. He/she can be removed from his office by guaranteed fundamental rights of citizens. In
the President on the recommendation of the order to scrutinize the legitimacy of
Parliament only. administrative action and the statutes, the
· Statement 2 is incorrect- A judge of the Constitution of India has given several powers to
Supreme Court can be removed from his office by the high courts and the Supreme Court of India.
an order of the President only. The President can To guard the rights of the public and implement
issue such removal orders only after an address the fundamental rights are the main objectives of
by Parliament has been presented to him in the judicial review.
same session for such removal. The address must
be supported by a special majority of each House Q.22) Ans: d
of Parliament. Exp:
· Statement 3 is correct- The Constitution
under Article 124(4) has provided for the Statement 1 is incorrect: There is no term
impeachment of judges on the grounds of proved prescribed for Council of Ministers. It remains in
misbehaviour or incapacity but the Constitution office as long as it enjoys the Confidence of Lok
has not given any mandate till date as to what Sabha.
constitutes 'misbehaviour' or 'incapacity'. Since Statement 2 is incorrect: The Head of Republic of
the commencement of the constitution no Judge India is President. The Council of Ministers is
of the Supreme Court has been impeached. The headed by the Prime Minister. Thus while the
process of impeachment has been kept complex President is de jure head, The Prime Minister is
to maintain the independence of the judiciary de facto head of State.
and to ensure safe tenure to the judges who are
imparting justice. Reference Que: Q26, Set D 2017
e
in

Q.21) Ans: d Q.23) Ans: b


nl

Exp: Exp:
l.o
ria

· Statement 1 is correct- Article 137 Statement 1 is correct: It is Constitutional for the


Constitution of India: Review of judgments or Council of Ministers that enjoys a majority in the
e
at

orders by the Supreme Court. Subject to the Lower house to advise the President to dissolve
cm

provisions of any law made by Parliament or any Lok Sabha anytime.


rules, the Supreme Court shall have power to Statement 2 is incorrect: The advice rendered by
ps

review any judgment pronounced or order made the Council of Minister cannot be inquired in
by it. court of law. It showcases the affinity in relation
.u
w

PRAYAS TEST 4 22
w
w
Contact us : info@onlyias.com

OnlyIAS Nothing Else Visit : dpp.onlyias.in


Contact : +91-7007 931 912

between President and Council of Ministers the residual legatee, that is, he looks after all
Statement 3 is incorrect: Article 72 of the those matters which do not fall within the
Constitution prescribes that the total number of purview of other secretaries. He also acts as
Ministers, including the Prime Minister, in the secretary, by rotation, of the zonal council and
Council of Ministers shall not exceed 15 exercises general supervision and control over
percent of the number of members of the House the entire state secretariat.
of the People. The 91st Amendment curbed the Statement 3 is correct: The chief secretary’s
PM's discretion to appoint any number of advice facilitates political decision-making. He
ministers. plays a supervisory role and acts as a resource
person in tendering advice.
Q.24) Ans: b Source:
Exp:
https://www.yourarticlelibrary.com/india-2/8-
Statement 1 is correct: - According to the functions-of-the-chief-secretary-of-a-state/46664
Constitution, the Chief Minister may be a
member of any of the two Houses of a state
legislature.
Statement 2 is incorrect: A person who is not a Q.26) Ans: c
member of the state legislature can be appointed Exp:
as Chief Minister for six months, within which Statement 1 is correct: Facilitating smooth
time, he should be elected to the state transaction of business in Ministries/
legislature, failing which he ceases to be the Chief Departments of the Government by assisting in
Minister decision-making in Government by ensuring
Statement 3 is correct: Since the Chief Minister Inter-Ministerial coordination, ironing out
is the head of the council of ministers, his differences and evolving consensus amongst
resignation or death automatically dissolves the Ministries/ Departments.
council of ministers. The resignation or death of Statement 2 is correct: It provides Secretarial
any other minister, on the other hand, merely assistance to the Cabinet and its Committees
creates a vacancy, which the Chief Minister may Statement 3 is correct: It helps in Monitoring,
or may not like to fill. Coordination and Promoting new policy initiatives
Statement 4 is incorrect: Chief Minister is Statement 4 is incorrect: It is the role of
the chairman of the State Planning Board and he the chief secretary of state but not cabinet
acts as a vice-chairman of the concerned zonal secretary to play supervisory role and as a
council by rotation, holding office for a period of resource person to the state government.
one year at a time.
Source: M.LaxmiKanth Q.27) Ans: c
e

Exp:
in

Q.25) Ans: d · Statement 1 is correct- The council of


nl

Exp: ministers exercises the executive authority of the


l.o

Statement 1 is correct: The chief secretary acts Union Government. But It is the cabinet which
ria

as the head of the state civil services, maintaining steers the ship of the state. Cabinet directs
morale of the civil servants. As a conscience- the council of ministers by taking decisions that
e
at

keeper of all state civil servants thus determining are binding on all ministers. Hence it can be said
cm

vacancies, appointments, transfers, placements, that functions of Council of minister is


seniority, promotions and retirement of public determined by cabinet.
ps

servants. · Statement 2 is incorrect- The Council of


Statement 2 is correct: The chief secretary acts as Ministers is collectively responsible to the Lower
.u
w

PRAYAS TEST 4 23
w
w
Contact us : info@onlyias.com

OnlyIAS Nothing Else Visit : dpp.onlyias.in


Contact : +91-7007 931 912

House of the Parliament .i.e. Lok Sabha Source: M.LaxmiKanth


· Statement 3 is correct- The Council of
Ministers doesn't meet as a body to transact the Q.30) Ans: c
government business. It has no collective Exp:
functions.
Statement 1 is incorrect: - A nominated
Q.28) Ans: b member of a House becomes disqualified for
Exp: being a member of the House if he joins any
political party after the expiry of six months from
· Statement 1 is incorrect: Article 88 says the date on which he takes his seat in the House.
that Every Minister shall have the right to speak This means that he may join any political party
and take part in the proceedings of either House, within six months of taking his seat in the House
any joint sitting of the houses and any committee without inviting this disqualification.
of parliament of which he is a member. but he Statement 2 is incorrect: An independent
shall not be entitled to vote. member of a House (elected without being set up
· Statement 2 is correct: Article 75(2) states as a candidate by any political party) becomes
that the ministers shall hold office during the disqualified to remain a member of the House if
pleasure of the President. All the ministers are he joins any political party after such election.
appointed by the President on the advice of the Source: M.LaxmiKanth
Prime Minister. Each minister has an individual
responsibility. Q.31) Ans: b
· Statement 3 is incorrect: The advice of the Exp:
Council of Ministers is binding on the President.
However, Article 74(2) barred courts from Statement 1 is incorrect: - If a member goes out
inquiring into the advice given by the Council of of his party as a result of a merger of the party
Ministers to the President. This provision with another party. A merger takes place
emphasizes the intimate and confidential when two- thirds of the members of the party
relationship between the president and the have agreed to such merger.
Council of Ministers. Statement 2 is correct: If a member, after being
elected as the presiding officer of the House,
Q.29) Ans: b voluntarily gives up the membership of his party
Exp: or rejoins it after he ceases to hold that office.
This exemption has been provided in view of the
Statement 1 is correct: - A member of a House dignity and impartiality of this office.
belonging to any political party becomes Source: M.LaxmiKanth
disqualified for being a member of the House, if
e

he voluntarily gives up his membership of such Q.32) Ans: c


in

political party. Exp:


nl

Statement 2 is incorrect: A member of a House


l.o

belonging to any political party becomes Statement 1 is incorrect: - Originally, the act
ria

disqualified for being a member of the House, if provided that the decision of the presiding
he votes or abstains from voting in such House officer is final and cannot be questioned in any
e
at

contrary to any direction issued by his political court. However, in Kihoto Hollohan case, it held
cm

party without obtaining prior permission of such that the presiding officer's decision is subject to
party and such act has not been condoned by the judicial review.
ps

party within 15 days. Statement 2 is correct: The presiding officer of a


House is empowered to make rules to give effect
.u
w

PRAYAS TEST 4 24
w
w
Contact us : info@onlyias.com

OnlyIAS Nothing Else Visit : dpp.onlyias.in


Contact : +91-7007 931 912

to the provisions of the Tenth Schedule. All such Exp:


rules must be placed before the House for 30
days. The House may approve or modify or · Statement 1 and 4 are correct: The law was
disapprove them. Further, he may direct that any supposed to and has certainly been able to curb
willful contravention by any member of such the evil of defection to a great extent. It has
rules may be dealt with in the same manner as a reduced corruption as well as non-developmental
breach of privilege of the House. expenditure incurred on a regular
Statement 3 is incorrect: The Presiding officer election. Political instability caused by frequent
cannot take up an anti-defection case on suo- and unholy change of allegiance on the part of
moto basis, but can take up only when he the legislators of our country has been contained
receives a complaint from a member of the to a very great extent. Moreover, it gives, for the
House. first time, a clear-cut constitutional
recognition to the existence of political parties.
Q.33) Ans: b · Statement 2 is incorrect: Despite the
Exp: positives, a very alarming trend of legislators
defecting in groups to another party in search of
· Statements 1 and 3 are correct: The 52nd greener pastures is visible. The recent examples
Amendment Act of 1985 provided for the of defection in state Assemblies (Goa, Karnataka
disqualification of the members of parliament etc.) and even in Rajya Sabha bear this out. This
and state legislatures on the ground of only shows that the law needs a relook in order
defection from one party to another. Since then to plug the loopholes it obviously has.
the law has been amended only once in · Statement 3 is incorrect: The anti-defection
2003. The 91st Amendment Act of 2003 law does not make
omitted a provision related to split in a political a differentiation between dissent and defection.
party. It curbs the legislator’s right to dissent and
· Statement 2 is incorrect: An independent freedom of conscience. It clearly puts party
member of a house becomes disqualified to bossism on a pedestal and sanctions tyranny of
remain a member of the house if he joins any the party in the name of the party discipline.
political party after such election. A nominated
member of a house becomes disqualified for Q.35) Ans: d
being a member of the house if he joins any Exp:
political party after the expiry of six
months from the date on which he takes his seat · Statement 1 is incorrect: The
in the house. disqualification on the ground of defection does
· Statement 4 is correct : Member of a house not apply if a member goes out of his party as a
belonging to any political party becomes result of a merger of the party with another
e

disqualified for being a member of the house party. A merger takes place when two-thirds of
in

the members of the parties have agreed to such a


nl

 if he voluntarily gives up his membership of such Merger.


l.o

political party or · Statement 2 is incorrect: The provision of


ria

 if he abstains from voting in such house contrary the 10th schedule pertaining to exemption from
to any direction issued by his political party disqualification in case of split by one third
e
at

without obtaining prior permission of such party members of the Legislature party has
cm

and search act has not been condoned by the been deleted by the 91st Amendment Act of
party within 15 days. 2003. It means that the defections have no
ps

protection on Grounds of splits.


Q.34) Ans: d · Statement 3 is correct: The disqualification
.u
w

PRAYAS TEST 4 25
w
w
Contact us : info@onlyias.com

OnlyIAS Nothing Else Visit : dpp.onlyias.in


Contact : +91-7007 931 912

on the ground of defection does not apply if a new cabinet advises the President not to give his
member, after being elected as the presiding assent to such bills.
officer of the house, voluntarily gives up the
membership of his party or rejoins it after he Suspensive Veto
ceases to hold that office. This exemption has The President exercises this veto when he returns
been provided in view of the dignity and a bill for reconsideration of the Parliament.
impartiality of this office. However, if the bill is passed again by the
· Statement 4 is correct: A nominated Parliament with or without amendments and
member of a house does not become disqualified again presented to the President, it is obligatory
for being a member of the house if he joins any for the President to give his assent to the bill.
political party before the expiry of six
months from the date on which he takes his seat Pocket Veto
in the house. In this case, the President neither ratifies nor
rejects nor returns the bill, but simply keeps the
Q.36) Ans: c bill pending for an indefinite period. This power
Exp: of the President not to take any action (either
positive or negative) on the bill is known as the
The veto power enjoyed by the executive in pocket veto. The President can exercise this veto
modern states can be classified into the power as the
following four types: The Constitution does not prescribe any time-
1. Absolute veto: withholding of assent to the bill limit within which he has to take the
passed by the legislature. decision with respect to a bill presented to him
2. Qualified veto: which can be overridden by the for his assent.
legislature with a higher majority.
3. Suspensive veto: which can be overridden by Q.37) Ans: d
the legislature with an ordinary majority. Exp:
4. Pocket veto: that is, taking no action on the bill
passed by the legislature. Statement 1 is Incorrect: He decides on questions
as to disqualifications of members of the
Of the above four, the President of India is Parliament, in consultation with the Election
vested with three—absolute veto, suspensive Commission not Speaker.
veto and pocket veto. There is no qualified veto Statement 2 is Incorrect: The President can seek
in the case of Indian President; it is possessed by advice from the Supreme Court on any question
the American President. of law or fact. However, the advice tendered by
the Supreme Court is not binding on the
The three vetoes of the President of India are President.
e

explained below: Statement 3 is Correct: The President has no veto


in

Absolute Veto power in respect of a Constitutional Amendment


nl

It refers to the power of the President to bill. The 24th Constitutional Amendment Act of
l.o

withhold his assent to a bill passed by the 1971 made it obligatory for the President to give
ria

Parliament. The bill then ends and does not his assent to a constitutional amendment bill.
become an act. Usually, this veto is exercised in
e
at

the following two cases: Q.38) Ans: d


cm

(a) With respect to private members’ bills Exp:


(b) With respect to the government bills when
ps

the cabinet resigns (after the passage of the bills · Statement 1 is incorrect: The resolution for
but before the assent by the President) and the the impeachment of the President is moved after
.u
w

PRAYAS TEST 4 26
w
w
Contact us : info@onlyias.com

OnlyIAS Nothing Else Visit : dpp.onlyias.in


Contact : +91-7007 931 912

14 days' notice but it should be signed by not less Parliament is superior and simultaneously the
than one-fourth members of the House. After President cannot promulgate ordinance, hence
the impeachment resolution is passed by the power of President to legislate by ordinance
a majority of two-third of the total membership is not parallel to the power of legislation
of that House, it is sent to another House, which · Statement 2 is incorrect: The President can
should investigate the charge. promulgate ordinance only when both the
· Statement 2 is incorrect: The president of Houses of Parliament are not in session or when
India can be removed from office by a process of either of the House is not in session or one House
impeachment for 'violation of the Constitution'. is in session. An ordinance made when both the
However, the Constitution does not define the Houses are in session is void.
meaning of the phrase 'violation of the · Statement 3 is correct; The President makes
Constitution'. Other than this he can also vacate an ordinance when he is satisfied that such
his office by resignation in writing under his hand circumstance does exist to take immediate
addressed to the Vice-President. action. Thus the satisfaction of the President is
Source: M. LAXMIKANTH, D.D. BASU important in making an ordinance.
· Statement 4 is incorrect: In the Cooper case
Q.39) Ans: c (1970), the Supreme Court held that the
Exp: President's satisfaction can be questioned in a
court on the ground of malafide.
· Statement 1 is correct: When the vacancy is Sources: M. LAXMIKANTH, D.D. BASU
going to be created by the expiration of the term
of sitting President, an election to fill the vacancy Q.41) Ans: b
must be completed before expiration of the term. Exp:
In case of any delay in the election, the outgoing
President continues to hold the office until his · Statement 1 is correct: As per Article 71 (a),
successor assumes the office. all the disputes and doubts in connection with
· Statement 2 is correct: In order to prevent election of President or Vice-President are
'interregnum', owing to any possible delay in enquired into and by the Supreme Court whose
conducting the election of a new President by any decision is exclusive and final.
reason, it is provided that the outgoing President · Statement 2 is incorrect: Election of the Vice
must continue to hold office, notwithstanding president cannot be challenged on the ground of
that his term has expired, until his successor any vacancy in the Electoral College which
enters upon his office. elected the Vice-President. Also if the election of
· Statement 3 is incorrect: The Vice-President the Vice-President is declared void by the
cannot act as the President after the expiration of Supreme Court, acts done by him prior to the
the term of the sitting President, as the outgoing date of such decision of the Supreme Court shall
e

President continues to hold office until his not be invalidated.


in

successor enters upon his office after a fresh · Statement 3 is correct and statement 4
nl

election. incorrect: Although Indian Vice-President is


l.o

Source: M. LAXMIKANTH, D.D. BASU modeled on the lines of American Vice-President,


ria

there is a difference. The American Vice-


Q.40) Ans: c President succeeds to the presidency when it falls
e
at

Exp: vacant, and remains President for the unexpired


cm

term of his predecessor. The Indian Vice-


· Statement 1 is incorrect: The President President, on the other hand, does not assume
ps

cannot promulgate ordinance when both Houses the office of the President when it falls vacant for
are in session. The legislative power of the unexpired term. He merely serves as an
.u
w

PRAYAS TEST 4 27
w
w
Contact us : info@onlyias.com

OnlyIAS Nothing Else Visit : dpp.onlyias.in


Contact : +91-7007 931 912

acting President until the new President is · Statement 4 is incorrect: The Prime Minister
elected as the leader of the Lower House, in this capacity
Source: M. LAXMIKANTH, D.D. BASU he/she can recommend the dissolution of the Lok
Sabha to the President at any time.
Q.42) Ans: b Source: D.D. BASU, M. LAXMIKANTH
Exp:
Q.44) Ans: b
· Statement 1 is correct: The electoral college Exp:
of Vice-President consists of both elected and
nominated members of the parliament (in case of · Statement 1 is correct: The Prime Minister is
President, only elected members). the leader of the Lower House in this capacity he
· Statement 2 is incorrect: It does not include advises the President with regard to summoning
the members of state legislative assemblies. The and proroguing of the sessions of the Parliament.
President is the head of the State and his power · Statement 2 is correct: As the leader of the
extends both to the administration by Centre as Lower House Prime minister announces policies
well as states. Consequently, it is necessary that of the government on the floor of the House.
in his election, not only members of Parliament · Statement 3 is incorrect: This power of
should play their part, but the members of the the Prime Minister is in relation with the
state legislatures should have a voice. President, in which he acts as a communication
· Statement 3 is incorrect: The electoral channel between the President and the council of
college of Vice-President does not consist of ministers.
Union Territory of Delhi and Puducherry (in case · Statement 4 is incorrect: This power is also
of President, elected members of legislative in relation with the President, the Prime Minister
assemblies of State and Union Territory of Delhi communicates to the President all decisions of
and Puducherry are included). the council of ministers relating to the
Source: D.D. BASU, M. LAXMIKANTH administration of the affairs of the union and
proposals of the legislation.
Q.43) Ans: d Source: M. LAXMIKANTH
Exp:
Q.45) Ans: c
· Statement 1 is correct: The Constitution Exp:
does not contain any specific procedure for the
selection and appointment of the Prime Minister. · Statement 1 is correct: Article 72 of the
Article 75 says only that the Prime Minister shall Constitution empowers the President to grant
be appointed by the President but does not pardons, the objective of conferring this power to
describe any procedure. the President is to keep the door open for
e

· Statement 2 is incorrect: The term of Prime correcting any judicial errors in the operation of
in

Minister is not fixed and he holds office during law.


nl

the pleasure of the President. However, it does · Statement 2 is incorrect: The pardoning
l.o

not mean that the President can dismiss the power of the President is not similar to the
ria

Prime Minister at any time. If The Prime Minister pardoning power of the Governor, as the
enjoys the majority support in the Lok Sabha, President can pardon even a death sentence and
e
at

he/she cannot be dismissed by the President. also any punishment or sentence by a court-
cm

· Statement 3 is correct: The Prime Minister martial but, the Governor cannot do so, the
as the head of the Union council of ministers, pardoning power of the Governor cannot extend
ps

hence he/she can bring about the collapse of the beyond the punishment or sentence prescribe to
council of ministers by resigning from office. a person against a state law.
.u
w

PRAYAS TEST 4 28
w
w
Contact us : info@onlyias.com

OnlyIAS Nothing Else Visit : dpp.onlyias.in


Contact : +91-7007 931 912

· Statement 3 is correct: Through pardoning Statement 4 is Correct: The Governor appoints


the President also affords relief from a sentence, the chairman and members of the State public
which the President regards as unduly harsh. service commission. However, they can be
· Statement 4 is incorrect: President exercises removed only by the president and not by a
pardoning power on the advice of the Union governor.
Cabinet.
Source: M. LAXMIKANTH Q.48) Ans: a
Exp:
Q.46) Ans: d
Exp: · Statement 1 is correct: The President may
make such provision as he thinks fit for the
Statement 1 is Incorrect: The Discretionary discharge of the functions of the governor in any
powers exercised by the Governor is not beyond contingency not provided in the Constitution, for
Judicial review. The Supreme Court, speaking example, the death of a sitting governor. Thus,
through a five-judge Bench in Nabam Rebia and the chief justice of the concerned state can be
Bamang Felix v. Deputy Speaker (2016) ruled appointed temporarily as the governor of that
that the discretionary power of the Governor is state.
extremely limited and entirely amenable to · Statement 2 is incorrect: The constitution
judicial review. does not lay down any grounds upon which the
Statement 2 is Incorrect: As the money bill is governor may be removed by the President. The
introduced with the Governor’s permission, he President can transfer a Governor from one state
cannot return the money bill for the to another state for the rest of the term.
reconsideration of the State Legislature. He can · Statement 3 is incorrect: A Governor holds
either give his assent, withhold his assent or office for a term of five years from the date on
reserve the bill for consideration of the president. which he enters upon his office. However, his
Source: https://www.thehindu.com/opinion/op- term is subject to the pleasure of the President
ed/governors-discretion/article25021561.ece but, the Supreme Court held that the pleasure of
the President is not Justifiable.
Q.47) Ans: d Source: M. LAXMIKANTH
Exp:
Q.49) Ans: d
Statement 1 is Incorrect: Usually, there is a Exp:
governor for each state, but the 7th
Constitutional Amendment Act of 1956 facilitated · Statement 1 is incorrect: Governor can
the appointment of the same person as a grant pardons, reprieves, respite and remissions
governor for two or more states. of punishment or suspend, remit and commute
e

Statement 2 is Incorrect: Supreme Court in 1979, the sentence of any person convicted of any
in

the office of governor of a state is not an offence against any law but, law relating to a
nl

employment under the Central government. It is matter to which the executive power of the state
l.o

an independent constitutional office and is not extends and not outside the jurisdiction of the
ria

under the control of or subordinate to the state executive power.


Central government · Statement 2 is correct: While appointing the
e
at

Statement 3 is Incorrect: The oath of office to judges of the high court of the concerned state
cm

the Governor is administered by the Chief justice the governor of that state is consulted by the
of the concerned state high court and in his President. The persons of the state judicial
ps

absence, the senior-most judge of that court service (other than district judges) are also
available( not the President) appointed by the Governor in consultation with
.u
w

PRAYAS TEST 4 29
w
w
Contact us : info@onlyias.com

OnlyIAS Nothing Else Visit : dpp.onlyias.in


Contact : +91-7007 931 912

the state high court and the State Public Service into several autonomous regions, if different
Commission. tribes are present in the autonomous district.
· Statement 3 is incorrect: He cannot pardon · Statement 3 is incorrect: The power to
a death sentence. Even if a state law prescribes declare an area to be scheduled area lies with the
for the death sentence, the power to grant President and not the Governor.
pardon lies with the President and not with the · Statement 4 is correct: The Governor is
governor. But, the governor can suspend, remit empowered to appoint a commission to examine
or commute a death sentence. and report on any matter relating to the
· Statement 4 is correct: All the administration of the autonomous districts or
appointments, postings and promotions of the regions. He may also dissolve a district or regional
district judges are done in consultation with the council on recommendation of the commission.
state high court. Source: M. LAXMIKANTH
Source: M. LAXMIKANTH
Q.52) Ans: a
Exp:
Q.50) Ans: a
Exp: · Statement 1 is correct: As the head of the
state council of ministers the Chief Minister can
· Statement 1 is correct: The discretionary allocate the portfolios among the ministers.
power of the Governor is greater than the · Statement 2 is correct: The Chief Minister
discretionary power of the President, as the can bring about the collapse of the council of
President has only situational discretion and not ministers by resigning from office. Since the Chief
constitutional discretion, while the Governor has Minister is the head of the council of the
both the discretion powers, for example he can ministers, his resignation or death automatically
use situational discretion to appoint the Chief dissolves the council of ministers.
Minister when no party has a clear-cut majority · Statement 3 is incorrect: It is In relation to
(same in case of the President). Some the Governor he/she communicates to the
constitutional discretion are reservation of bills governor of the state all the decisions of the
for consideration of the president, seeking council of the ministers relating to the
information from the chief minister with regard administration of the affairs of the state and
to administrative and legislative matters of state. proposals or legislation
· Statement 2 is incorrect: The Governor has · Statement 4 is incorrect: As the leader of
both situational as well as constitutional the house, the Chief Minister can announce the
discretion, while the President has only government policies on the floor of the house.
situational discretion. This power of the Chief Minister is in relation
Source: M. LAXMIKANTH, D.D. BASU with the State Legislature and not with the
e

Council of Ministers.
in

Q.51) Ans: b Source: M. LAXMIKANTH


nl

Exp:
l.o

Q.53) Ans: b
ria

· Statement 1 is correct: The Governor is Exp:


empowered to organize and reorganize the
e
at

autonomous districts. Thus, he can increase or · Statement 1 is correct: Article 163, states
cm

decrease their areas or change their names or that there shall be a council of ministers with the
define their boundaries and so on. Chief Minister as the head to aid and advise the
ps

· Statement 2 is correct: The Governor is Governor on the exercise of his functions, except
empowered to divide the autonomous district insofar as he is required to exercise his functions
.u
w

PRAYAS TEST 4 30
w
w
Contact us : info@onlyias.com

OnlyIAS Nothing Else Visit : dpp.onlyias.in


Contact : +91-7007 931 912

or any of them at his discretion. Q.55) Ans: b


· Statement 2 is incorrect: Article 167, states Exp:
that it shall be the duty of the Chief Minister to
communicate to the Governor of the state all · Statement 1 is correct: For ensuring the
decisions of the council of ministers relating to independence of the Supreme Court it is
the administration of the affairs of the state and bestowed with the power to punish for its
proposals for legislation and to furnish such contempt. The Supreme Court can punish any
information relating to the administration of the person for its contempt. Its action and decisions
affairs of the state and proposals for legislation as cannot be criticized and opposed by anybody.
the Governor may call for. The Contempt of Court may be civil or criminal.
· Statement 3 is incorrect: The Council of However, reasonable criticism of the judiciary act
Ministers shall be collectively responsible to does not amount to contempt of court.
the Legislative Assembly of the State. The · Statement 2 is incorrect: The jurisdiction of
ministers are jointly and severally responsible to the Supreme Court cannot be curtailed. The
the Legislature. He or they, is or are, publicly Parliament is not authorised to curtail the
accountable for the acts or conducts in the jurisdiction and powers of the Supreme Court.
performance of duties. However, the Parliament can extend the same.
· Statement 3 incorrect: Article 143 of the
Source: M. LAXMIKANTH, D.D. BASU Constitution of India confers upon the Supreme
Court advisory jurisdiction. The president can
Q.54) Ans: c seek the opinion of the Supreme Court in the two
Exp: categories of matters- on the question of law fact
of public importance and on any dispute arising
Statement 1 is correct: The governor is an out of any pre constitutional agreement treaty
integral part of the state legislature. In that etc or other similar instruments.
capacity, he can appoint any member of the state
legislative assembly to preside over proceedings Q.56) Ans: d
when the offices of both the Speaker and the Exp:
Deputy Speaker fall vacant. Similarly, he can
appoint any member of the state legislative · Statement 1 is incorrect: The writ
council to preside over its proceedings when the jurisdiction of the Supreme Court is not exclusive.
offices of both Chairman and the Deputy The high courts are also empowered to issue
Chairman fall vacant. writs for the enforcement of fundamental rights.
Each High Court has power to issue to any person
Statement 2 is correct: Article 192 (Decision on within its jurisdiction directions/ writs for
question as to disqualifications of members) enforcement of Fundamental Rights and for any
e

1. If any question arises as to whether a member other purpose.


in

of a House of the Legislature of a State has · Statement 2 is incorrect: An aggrieved


nl

become subject to any of the disqualifications person can directly go to the Supreme Court for
l.o

mentioned in article 191, the question shall be the enforcement of the fundamental rights in this
ria

referred for the decision of the Governor and his scenario the Supreme Court has original
decision shall be final. jurisdiction’. Therefore, there is no need for an
e
at

2. Before giving any decision on any such aggrieved person to compulsorily approach the
cm

question, the Governor shall obtain the opinion High Court first for the enforcement of the
of the Election Commission and shall act Fundamental rights.
ps

according to such opinion. · Statement 3 is correct: Article 32 of the


Indian Constitution empowers the Supreme Court
.u
w

PRAYAS TEST 4 31
w
w
Contact us : info@onlyias.com

OnlyIAS Nothing Else Visit : dpp.onlyias.in


Contact : +91-7007 931 912

to issue writs to enforce only the fundamental · Statement 2 is incorrect: The constitution
rights against a person or government has not prescribed a minimum age for
throughout the territory of India. Whereas, appointment as judge of high court. There are
Article 226 empowers the High Courts of India to some qualifications required for a person to be
issue writs for the enforcement of appointed as the Judge of the High Court-
the fundamental rights as well as for other · Statement 3 is correct: A judge of high
purposes like the enforcement of an ordinary court can be removed from his office by an order
legal right. High courts can issue writs against a of the president; the president can issue the
person residing, government or authority located removal order only after an address by the
within its territorial jurisdiction Or Outside its Parliament has been presented to him in the
territorial jurisdiction if the cause of action arises same session for such removal. The President of
within its territorial jurisdiction. India can remove a Judge of the High Court, from
his office only if each house of the parliament
Q.57) Ans: a passes a resolution by a special majority of its
Exp: members in each house requesting him to
remove the judge.
· Pair 1 is correct: Certiorari means to
‘certify’. Certiorari Is issued by a Higher Court to a Q.59) Ans: b
Lower court / Tribunal to transfer a case pending Exp:
with the latter to itself or to squash the order of
the latter in a case. it is issued on the grounds of · Statement 1 is incorrect: The 42nd
excess of Jurisdiction or lack of Jurisdiction or amendment act of 1976 curtailed the Judicial
error of law. review power of the High Court. It debarred the
· Pair 2 is incorrect: Mandamus is issued by a high court from considering the constitutional
court to the public official asking him to perform validity of any Central law. But the 43rd
his official duties; it is in the nature of a Amendment Act of 1977 restored the original
command. it can also be issued against any public position.
body and inferior Court. · Statement 2 is correct: In addition to
· Pair 3 is correct: Habeas Corpus writ is used the appellate jurisdiction and supervisory
to release a person who has been unlawfully jurisdiction over the subordinate courts, the High
detained or imprisoned. The Court directs the Court has administrative control over them too.
person so detained to be brought before it to its laws are binding on all subordinate courts
examine the legality of his detention. If the Court functioning within its territorial jurisdiction in the
concludes that the detention was unlawful, then same sense as the law declared by the supreme
it directs the person to be released immediately. court is binding on all Courts in India.
· Statement 3 is correct: As a court of record,
e

Q.58) Ans: c the high court has the power to review and
in

Exp: correct its own judgement / order / decision even


nl

though in this case no specific power of review is


l.o

· Statement 1 is incorrect: The Judges of the conferred on the high courts by the
ria

High Court are appointed by the president. The constitution.


chief justice is appointed by the president after
e
at

consultation with the Chief Justice of India and Q.60) Ans: a


cm

the governor of the state concerned. For the Exp:


appointment of other judges, the president also
ps

consults the Chief Justice of the concerned High · Statement 1 is correct: the Parliament can
Court. extend the jurisdiction of the High Court to any
.u
w

PRAYAS TEST 4 32
w
w
Contact us : info@onlyias.com

OnlyIAS Nothing Else Visit : dpp.onlyias.in


Contact : +91-7007 931 912

Union Territory or exclude the jurisdiction of the directly mentioned in constitution. The Collegium
High Court from any union territory. System is that under which appointments and
· Statement 2 is incorrect: The salaries and promotion and transfer of the judges of the
allowances of the judges, the salaries allowances Supreme Court are decided by a forum which
and pensions of the staff as well as administrative consists of the Chief Justice of India plus four of
expenses of the High Court are charged to the the senior-most judges of the Supreme Court.
consolidated fund of the state. However, the · Statement 2 is correct: When there is a lack
pension of a High Court judge is charged on the of quorum of the permanent judges to hold or
consolidated fund of India. continue any session of the Supreme Court, the
· Statement 3 is correct: In Chandra Kumar Chief Justice of India can appoint a judge of a
case 1997, the Supreme Court ruled that the writ High Court as an ad hoc judge of the Supreme
jurisdiction of both the High Court under article Court for a temporary period. He can do so only
226 and that of the supreme court under article after consultation with the Chief Justice of the
32 Constitutes a part of the basic structure of the High Court concerned and with the previous
constitution. consent of the president. The judge so appointed
should be qualified for appointment as a judge of
Q.61) Ans: d the Supreme Court.
Exp: · Statement 3 is correct: Master of Roster
refers to the privilege of the Chief Justice to
· Option 1 is correct: Article 226 of the constitute Benches to hear cases. No Judge can
Constitution empowers a high court to issue writs take up the matter on this own, unless allocated
including habeas corpus, mandamus, certiorari, by the Chief Justice of India, as he is the master of
prohibition and quo warranto for the the roster.”
enforcement of the fundamental rights of the
citizens and for any other purpose. Q.63) Ans: c
· Option 2 is correct: Disputes relating to Exp:
election of members of parliament are under
original jurisdiction of the high court.  Statement 1 is incorrect: A High court has the
Further Article 323 B empowers the appropriate power of superintendence over all courts and
legislature (Parliament or a state legislature) to tribunals functioning in its territorial jurisdiction
establish a tribunal for the adjudication of (except military courts or tribunals)
election disputes.  Statement 2 is incorrect: As a court of record a
· Option 3 is correct: Contempt of court also high court also has the power to review and
comes under original jurisdiction of high court. In correct judgement or order or decision, even
India, the contempt law is mainly governed by though no specific power of review is conferred
the Contempt of Courts Act, 1971 as it empowers on it by the constitution. The SC on the other
e

the court to punish acts of contempt because of hand has been specifically conferred with the
in

which the Supreme Court and High Courts, by the power of review by the constitution.
nl

virtue of being courts of record hold inherent  Statement 3 is correct: Judicial review is the
l.o

jurisdiction to punish for contempt of court. power of a high court to examine the
ria

constitutionality of legislative enactments and


Q.62) Ans: b executive orders of both central and state
e
at

Exp: governments.
cm

· Statement 1 is incorrect: The collegium


Extraedge by Onlyias
ps

system has its genesis in a series of Supreme


The phrase’ Judicial review’ has
Court judgments called the ' Judges Cases' not
.u
w

PRAYAS TEST 4 33
w
w
Contact us : info@onlyias.com

OnlyIAS Nothing Else Visit : dpp.onlyias.in


Contact : +91-7007 931 912

nowhere been used in the Appointment of persons (other


constitution, the provisions of than district judges) to the judicial
Articles 13 and 226 explicitly services of a state are made by the
confer the power of judicial governor of the state after
review on a High Court. consultation with the state public
service commission and the high
Source of reference Polity Laxmikant court

Q.64) Ans: b
Exp: Source of reference Laxmikant , DD basu
Statement 1 is incorrect: The salaries,
allowances, privileges, leave of the judges of a Q.66) Ans: b
high court are determined from time to time by Exp:
the Parliament. salaries, allowances of the high
court judges are charged on the consolidated · Statement 1 is incorrect:The President can
fund of state and are non-votable by the state transfer a judge from one high court to another
legislature. It should be noted here that the after consulting the Chief Justice of India (not
pension of a high court judge is charged on the Governor).In third judge case supreme court said
consolidated fund of India and not the state. that in case of transfer of high court judges, the
Statement 2 is correct:The Constitution does not chief justice of India should consult, in addition to
contain detailed provisions with regard to the the collegium of the four senior most judges of
jurisdiction and the powers of a high court. It only supreme court, the chief justice of the two
lays down that the jurisdiction and powers of a concerned high court. On transfer he is entitled
high court are to be the same as immediately to receive in addition to his salary such
before the commencement of the Constitution. compensatory allowance as may be determined
by Parliament.
Source of reference Polity Laxmikant · Statement 2 is correct: In 1994 the SC held
that judicial review is necessary to check
Q.65) Ans: a arbitrariness in transfer of judges but only the
Exp: judge who is transferred can challenge it.

Statement 1 is correct: The appointment, posting Q.67) Ans: d


and the promotion of the district judges in the Exp:
state are made by the Governor of the state in
consultation with the high court. A person to be · Statement 1 is incorrect The constitution of
appointed as a district judge should not be India provides for a high court for each state , but
e

the seventh amendment act of 1956 authorised


in

already in the service of the central and state


government the Parliament to establish a common high court
nl

Statement 2 is incorrect: For eligibility he/she for two or more states or for two or more states
l.o

should have been an advocate or a pleader for and a Union territory.


ria

seven years and not 10 years · Statement 2 is correct The Parliament can
extend the jurisdiction of a high court to any
e

Statement 3 is incorrect: He/she should be


at

recommended by the high court for appointment union territory or exclude the jurisdiction of a
cm

and not SC high court from any union territory.


· Statement 3 is correct: A person appointed
ps

as a judge of a high court , before entering upon


Extra edge by Onlyias his office, has to make and subscribe an oath or
.u
w

PRAYAS TEST 4 34
w
w
Contact us : info@onlyias.com

OnlyIAS Nothing Else Visit : dpp.onlyias.in


Contact : +91-7007 931 912

affirmation before the Governor of the state or · Statement 2 is correct: The amendment also
some person appointed by him for this purpose. added article 75 (1B) according to which A
member of either House of Parliament belonging
Source of reference Laxmikant to any political party who is disqualified for being
a member of that House under the Tenth
Q.68) Ans: c Schedule shall also be disqualified to be
Exp: appointed as a Minister
· Statement 3 is correct: The 91st
· Statement 1 is correct. A High court is amendment also added article 361B according to
primarily a court of appeal. It hears appeals which a member of a House belonging to any
against the judgements of subordinate courts political party who is disqualified for being a
functioning in its territorial jurisdiction. It has member of the House under the Tenth Schedule
appellate jurisdiction in both civil and criminal shall also be disqualified to hold any
matters. Hence the appellate jurisdiction of a remunerative political post.
high court is wider than its original jurisdiction.
· Statement 2 is correct: Judicial review is the o The expression “remunerative political post”
power of a high court to examine the means any office—
constitutionality of legislative and executive  under the Government of India (State) where the
orders of both the central and state salary or remuneration for such office is paid out
governments. Though the phrase 'judicial review’ of the public revenue of the Government of
has nowhere used in the constitution, the India(State)
provisions of Articles 13 and 226 explicitly confer  under a body, whether incorporated or not,
the power of judicial review on a high court. which is wholly or partially owned by the
Government of India (State) and the salary or
remuneration for such office is paid by such body,
Extra edge by Onlyias
except where such salary or remuneration paid is
Like the Supreme court, the high
compensatory in nature.
court has been vested with quite
extensive and effective powers.it Q.70) Ans: a
is the highest court of appeal in Exp:
the state. At present a HC enjoys
the jurisdiction and powers some · Statement 1 is incorrect: It is the presiding
of which includes : original officer (chairman or speaker, as the case may be)
jurisdiction, writ jurisdiction, of a house who is empowered to make rules to
appellate jurisdiction, a court of give effect to the provisions of the tenth
record , power of judicial review. schedule. All such rules must be placed before
e

the house for 30 days. The house may approve or


in

Q.69) Ans: b modify or disapprove them further. The presiding


nl

Exp: officer may also direct that any wilful


l.o

contravention by any member of such rules may


ria

· Statement 1 is be dealt with in the same manner as a breach of


incorrect: The 91st amendment to the privilege of the house.
e
at

constitution added article 75 (1A) according to · Statement 2 is incorrect: According to the


cm

which the total number of Ministers, including rules made, the presiding officer can take a
the Prime Minister, in the Council of Ministers defection case only when he receives a
ps

shall not exceed fifteen per cent of the total complaint from a member of the house. Before
number of members of the House of the People. taking the final decision, she must give the
.u
w

PRAYAS TEST 4 35
w
w
Contact us : info@onlyias.com

OnlyIAS Nothing Else Visit : dpp.onlyias.in


Contact : +91-7007 931 912

member a chance to submit an. She may also


refer the matter to the committee of · Statement 1 is incorrect: Chief- Secretary
privileges for enquiry. acts as an advisor to the CM. He/ She acts as a
· Statement 3 is Correct: The anti defection secretary to the state cabinet. Chief Secretary is
act empowers the presiding officer of the house the administrative head of the cabinet secretariat
(chairman or the speaker) to decide on any and attends the meeting of the cabinet and its
question regarding the disqualification of a sub-committees.
member arising out of defection. Originally the · Statement 2 is correct: The chief secretary
act provided that the decision of the presiding also acts as the residual legatee which means he
officer is final and cannot be questioned in any looks after all those matters which do not fall
Court. within the purview of other secretaries.
o However, The Supreme court, in the Kihoto · Statement 3 is correct: The civil services
Hollohan versus Zachillhu case (1992), observed board is headed by the Chief Secretary of a state
that the suspicion of bias on the Speaker’s role and the board has senior most additional chief
could not be ruled out. It hence declared this secretary or chairman, Board of Revenue,
provision unconstitutional on the ground that it Financial Commissioner or an officer of
seeks to take away the jurisdiction of the equivalent rank and status as member.
Supreme Court and the high courts. The decision
of the presiding officer is thus subject to judicial Q.73) Ans: d
review. Exp:

Q.71) Ans: d · Statement 1 is incorrect: In 1971, the


Exp: Supreme Court held that even after the
dissolution of the Lok Sabha, the Council of
· Statement 1 is correct: The Cabinet Ministers does not cease to hold office. Hence,
Secretary is the top-most executive official and even when the Lok Sabha is dissolved, the
senior-most civil servant of the Government of President continues to work as per the aid and
India. She/he is the ex-officio head of the Civil advice of previous council of ministers till the new
Services Board and all civil services under the COM assumes office
rules of business of the government. She/he is · Statement 2 is incorrect: The cabinet is an
the senior-most cadre post of the Indian advisory body and a part of the Council of
Administrative Service. Minister it directs the Council of Ministers by
· Statement 2 is correct: She/He ranks on the taking policy decisions which are binding on all
same level as the Attorney General of India in the ministers. It exercises control over hire
order of precedence. Both are in the 11th rank in appointments like constitution authorities and
the table of precedence is - The Attorney General senior Secretariat administrators.
e

of India, Cabinet Secretary, Lieutenant governors · Statement 3 is correct: Article 74(1) states
in

with their respective union territories. that there shall be a Council of Ministers with a
nl

· Statement is correct: A cabinet Secretary is Prime Minister at the head to aid and advise the
l.o

appointed for a fixed tenure of two years. The President. However, the president may require
ria

central government may give an extension in the Council of Ministers to reconsider such advice
service for a further period not exceeding three and the president shall act in accordance with the
e
at

months, beyond the period of four years to a advice tendered after such reconsideration.
cm

cabinet secretary.
Q.74) Ans: c
ps

Q.72) Ans: a Exp:


Exp: · Pair 1 is correct: The principle of collective
.u
w

PRAYAS TEST 4 36
w
w
Contact us : info@onlyias.com

OnlyIAS Nothing Else Visit : dpp.onlyias.in


Contact : +91-7007 931 912

responsibility finds a place in Art. 75(3). It is the Sixth Schedule, contains special provisions for the
duty of every minister to stand by cabinet administration of tribal areas in the four north-
decisions and support them both within and eastern states of Assam, Meghalaya, Tripura and
outside the parliament.This also means that the Mizoram.
entire Council of ministers is a team that sinks or Statement 2 is incorrect: The tribal areas in the
swims together. So if the Lok Sabha passes a no four states of Assam, Meghalaya, Tripura and
confidence motion against the Council of Mizoram have been constituted as autonomous
ministers then all have to resign. districts. District and regional councils can make
· Pair 2 is incorrect: There is no such thing as laws on certain specified matters like land,
the Parliamentary Responsibility of Ministers forests, canal water, shifting cultivation, village
where the Duty of the Ministers to hold the administration, inheritance of property, marriage
Parliament sessions diligently. and divorce and social customs. But they do not
· Pair 3 is correct: Article 75 and Article164 fall outside the executive authority of the state
contains the principle of individual responsibility. concerned.
It states that the Ministers hold office during the Statement 3 is correct: The governor is
pleasure of the president / Governor . empowered to organise and re-organise the
autonomous districts. Thus, he can increase or
Q.75) Ans: b decrease their areas or change their names or
Exp: define their boundaries and so on.

Statement 1 is correct: The Fifth Schedule of the Q.77) Ans: b


Constitution deals with the administration of Exp:
Scheduled Areas and Scheduled Tribes residing in
any State other than the States of Assam, Statement 1 is incorrect: Each autonomous
Meghalaya, Tripura and Mizoram. The President district has a district council consisting of 30
is empowered to declare an area to be a members, of whom four are nominated by the
scheduled area. governor and the remaining 26 are elected on the
Statement 2 is basis of adult franchise. The elected members
incorrect: The President can increase or hold office for a term of five years (unless the
decrease its area or change its boundary lines or council is dissolved earlier) and nominated
make fresh orders for such re-designation on an members hold office during the pleasure of the
area in consultation with the governor of the governor.
state concerned. Statement 2 is correct: The autonomous district
Statement 3 is incorrect: Each state having is empowered to assess and collect land revenue
scheduled areas has to establish a tribes advisory and to impose certain specified taxes. Similar
council to advise on welfare and advancement of Powers are also available with regional councils.
e

the scheduled tribes. A similar council can also Statement 3 is correct: The district and regional
in

be established in a state having scheduled tribes councils administer the areas under their
nl

but not scheduled areas if the president so jurisdiction. They can make laws on certain
l.o

directs. specified matters like land, forests, canal water,


ria

shifting cultivation, village administration, and


Source: Laxmikanth inheritance of property, marriage and divorce,
e
at

social customs. But all such laws require the


cm

Q.76) Ans: b assent of the governor.


Exp: Source: laxmikanth
ps

Reference question: Question on use of minor


Statement 1 is incorrect: The Constitution, under
.u
w

PRAYAS TEST 4 37
w
w
Contact us : info@onlyias.com

OnlyIAS Nothing Else Visit : dpp.onlyias.in


Contact : +91-7007 931 912

minerals under 6th schedule Q.80) Ans: a


Exp:
Q.78) Ans: d
Exp: · Statement 1 is correct: The president is
empowered to declare an area to be a scheduled
Autonomous District councils: area. He can also increase or decrease its
· As per the Sixth Schedule, the four states area, alter its boundary lines, rescind such
viz. Assam, Meghalaya, Tripura and designation or make fresh orders for such re-
Mizoram contain the Tribal Areas which designation on area in consultation with the
are technically different from the Scheduled governor of the state concerned.
Areas (5th Schedule). · Statement 2 is correct: The executive power
· Though these areas fall within the executive of a state extends to the scheduled areas therein.
authority of the state, provision has been made But the governor has a special responsibility
for the creation of the District Councils and regarding such areas. He has to submit a report
regional councils for the exercise of the certain to the president regarding the administration of
legislative and judicial powers. such areas, annually or whenever so required by
· The Governor is empowered to : the president. The executive power of the Centre
extends to giving directions to the states
o Organise or Reorganise any area of autonomous regarding the administration of such areas.
district. · Statement 3 is incorrect: Each state having
o increase the area of any autonomous district a scheduled area has to Mandatorily establish
o diminish the area of any autonomous district. the tribes Advisory Council to advise on welfare
o alter the name of any autonomous district. and advancement of the scheduled tribes.
o define the boundaries of any autonomous It consists of 20 members, three-fourth of whom
district. are to be the representatives of the scheduled
tribes in the state legislative assembly. A similar
Q.79) Ans: b Council can also be established in a state having
Scheduled Tribes, but not schedule areas
Exp: therein,if the president so directs.

Q.81) Ans: a
Fifth schedule of the Indian Constitution: Exp:
· According to article 244 of part X of the
constitution, the provisions of the Fifth Schedule Governor and the Fifth schedule:
shall apply to the administration and control of · The governor can direct that any particular
the Scheduled Areas and Scheduled Tribes in any act of Parliament or the state legislature does
e

State other than the States of Assam, not apply to a scheduled area or apply with
in

Meghalaya, Tripura and Mizoram. specified modifications and exceptions.


nl

· The Philosophy behind the schedule areas is · He/she can also make regulations for the
l.o

that they are inhabited by aboriginals who are peace and good government of a scheduled area
ria

socially and economically backward and hence after consulting the tribes advisory council. Such
special efforts need to be made to improve their regulations may prohibit or restrict the transfer of
e
at

condition. Therefore the whole of the normal land by or among members of the scheduled
administrative machinery operating in the state
cm

tribes, regulate the allotment of land to members


is not extended to the schedule areas and of the scheduled tribes and regulate the business
ps

the central government has somewhat greater of money-lending in relation to the scheduled
responsibility for this area tribes.
.u
w

PRAYAS TEST 4 38
w
w
Contact us : info@onlyias.com

OnlyIAS Nothing Else Visit : dpp.onlyias.in


Contact : +91-7007 931 912

· Also, a regulation may repeal or amend any member (against whom the complaint has been
act of Parliament or the state legislature, which is made) a chance to submit his. He may also refer
applicable to a scheduled area. But, all such the matter to the committee of privileges for
regulations require the assent of the president. inquiry. Hence, defection has no immediate and
automatic effect. Decision of the Presiding officer
Q.82) Ans: d is subject to Judicial review
Exp:
Statement 2 is correct: The presiding officer of a
· Statement 1 is incorrect: Provisions related House is empowered to make rules to give effect
to administration of the scheduled tribes ( Fifth to the provisions of the Tenth Schedule
Schedule and Sixth Schedule of the Indian (provisions for anti-defection)
Constitution ) can be subjected to changes and
amendments by a simple majority of the Q.84) Ans: b
Parliament and hence do not require the Exp:
consent of the state legislature
· Statement 2 is correct: According to article Statement 1 is incorrect: Disqualification will not
339 of the Constitution of India, The President bar a member from contesting any future
may at any time appoint a Commission to report election during the remaining period of the
on the administration of the Scheduled Areas and house.
the welfare of the Scheduled Tribes in the States.
Hence, a commission was appointed in 1960. It Statement 2 is correct: As per provision added
was headed by U.N.Dhebar and submitted its by 91st amendment act a member of either
report in 1961. After four decades, the second house of Parliament belonging to any political
commission was appointed in 2002 under the party who is disqualified on the ground of
chairmanship of Dilip Singh Bhuria. It submitted defection shall also be disqualified to be
its report in 2004 appointed as a minister.
· Statement 3 is correct: The acts of
Parliament or the state legislature do not apply Statement 3 is correct: He shall also be
to autonomous districts and autonomous disqualified to hold any remunerative political
regions or apply with specified modifications post.
and exceptions. The power of direction, in this
regard, lies either with the president or governor. Q.85) Ans: a
Thus, in the case of Assam, it lies with Exp:
the Governor, both in respect of acts of
Parliament or state legislature. In the case · Statement 1 is correct: The Prime minister is
of Meghalaya, Tripura and Mizoram, it lies with appointed by the President and others ministers
e

the president in respect of acts are appointed by the president on the advice of
in

of Parliament and governor in respect of acts the PM. The ministers hold office during the
nl

of state legislature. pleasure of the president and they are


l.o

collectively responsible to the Lok sabha.


ria

Q.83) Ans: b · Statement 2 is incorrect In Article 74


Exp: constitution mentions that council of ministers to
e
at

aid and advise the president. The advice tendered


cm

Statement 1 is incorrect: the presiding officer can by the ministers to the president shall not be
take up a defection case only when he receives a inquired into any court.
ps

complaint from a member of the House. Before · Statement 3 is incorrect The salaries and
taking the final decision, he must give the allowances of ministers are determined by the
.u
w

PRAYAS TEST 4 39
w
w
Contact us : info@onlyias.com

OnlyIAS Nothing Else Visit : dpp.onlyias.in


Contact : +91-7007 931 912

Parliament (and not by the President) from time Exp:


to time and they receive the salaries and
allowances that are payable to a member of · Statement 1 is correct: Cabinet is a smaller
Parliament. body consisting of 15 to 20 ministers.it includes
the cabinet ministers only. Thus it is a part of the
Council of ministers. It supervises the
Extraedge by Onlyias
implementation of its decisions by the council of
The Principle of collective
ministers.
responsibility as mentioned in Article
· Statement 2 is incorrect: It was inserted in
75(CoM is collectively responsible to
Article 352 of the constitution in1978 by the 44th
LoK Sabha) forms the bedrock of the
constitutional amendment act. Thus it did not
Parliamentary system of
find a place in the original text of the
government.
constitution.
· Statement 3 is correct: The Cabinet is the
Source of reference Polity Laxmikant highest decision making authority in our politico-
administrative system. It is an advisory body to
Q.86) Ans: b the president and its advice is binding on them. It
Exp: is also the supreme executive authority in the
central government.
· Statement 1 is incorrect In Britain, every
order of the King for any public act is Source of reference Laxmikant Polity
countersigned by a minister. The minister would
be held responsible and would be liable in the Q.88) Ans: b
court If the order is in violation of any law. In Exp:
India, on the other hand, there are no provisions
in the constitution for the system of legal Statement 1 is incorrect: The Prime minister is
responsibility of a minister. appointed by the President, while the other
· Statement 2 is correct. In 1971, the ministers are appointed by the President on the
Supreme Court held that even after the advice of the Prime Minister. A minister who is a
dissolution of Lok sabha the council of ministers member of one house of Parliament has the right
does not cease to hold office. Article 74 is to speak and to take part in the proceedings, of
mandatory and therefore, the president cannot the other house also, but he can vote only in the
exercise the executive power without the aid and house of which he is a member.
advice will be unconstitutional. Statement 2 is correct: The ministers hold office
during the pleasure of the Governor. All executive
action of the Government of a state shall be
e

Extraedge by Onlyias
expressed to be taken in the name of the
in

The Constitution of India provided


Governor.
nl

for a Parliamentary form of


Statement 3 is correct: The constitution does not
l.o

government in which a council of


specify the exact size of the state council of
ministers headed by the Prime
ria

ministers or ranking of ministers. They are


Minister is the real executive
determined by the chief minister according to the
e

authority.
at

requirement and demand of the situation.


cm

Source of reference Polity Laxmikant


Extraedge by Onlyias
ps

The Governor is the nominal


Q.87) Ans: c
.u
w

PRAYAS TEST 4 40
w
w
Contact us : info@onlyias.com

OnlyIAS Nothing Else Visit : dpp.onlyias.in


Contact : +91-7007 931 912

executive authority ( de Extradedge by Onlyias


jure executive) and the chief The Chief secretary acts as a
minister is the real executive secretary to the state cabinet.He
authority(de facto executive). acts as the head of state civil
services.
Source of reference Polity Laxmikant
Source if reference
Q.89) Ans: d https://cabsec.gov.in/
Exp:
Q.91) Ans: c
Statement 1 is correct: They are appointed Exp:
among the members of the legislative assemblies
in the state. They often hold the rank of minister Statement 1 is correct: The post of Chief
of state and are appointed by the chief minister Secretary is the senior most position held in the
of the state concerned. civil services of the state or Union territory. He
Statement 2 is correct: They are ruling party acts as the head of state civil services. Important
members to assist ministers. They have no cases pertaining to transfers, appointment,
department in their control and are attached to and promotions of senior state civil servants goes
senior ministers. They are regarded for all to his/her desk.
practical purposes and having access to all files Statement 2 is incorrect Cabinet secretary (and
and documents. not chief secretary) is the administrative head of
the Union cabinet secretariat. Chief Secretary is
Source of reference the head of State cabinet secretariat whose
https://www.thehindu.com/news/national/Is- functions include secretarial assistance,
the-appointment-of-Parliamentary-Secretary- implementation of decisions and policy
unconstitutional/article14424083.ece coordination
Statement 3 is correct: The Chief secretary also
Q.90) Ans: a acts as the Principal advisor to the chief minister
Exp: on all the matters of State administration.

Statement 1 is correct: The Cabinet secretariat is


Extraedge by onlyias
responsible for the administration of the GoI
Chief secretary is the principle
(Transaction of Business rules),1961 and GoI
channel of communication
(Allocation of business) rules, 1961 facilitating
between the concerned state
smooth transaction of business in
e

government and the Central


ministries/departments. It assists in decision
in

Government.
making in government by ensuring inter-
nl

ministerial coordination and evolving consensus.


l.o

Statement 2 is incorrect: The cabinet secretariat Source of reference


ria

functions directly under the Prime Minister (and https://www.gktoday.in/upsc-questions/what-


not Home Minister) are-the-main-functions-of-the-chief-secretary-
e
at

Statement 3 is incorrect: The administrative head what-are-recent-arguments-against-the-post-of-


cm

of the Secretariat is the Cabinet Secretary (and chief-secretary/


not Chief Secretary) who is also the ex-officio
ps

Chairman of the Civil Services Board. Q.92) Ans: b


.u

Exp:
w

PRAYAS TEST 4 41
w
w
Contact us : info@onlyias.com

OnlyIAS Nothing Else Visit : dpp.onlyias.in


Contact : +91-7007 931 912

Q.94) Ans: b
· Statement 1 is incorrect: The Vice-President Exp:
occupies the second highest office in the country
The office of Vice President is modelled on  Statement 1 is incorrect
the lines of the American Vice-President.
· Statement 2 is correct: The resolution for A governor holds office for a term of five
removal of the Vice President can be introduced years from the date on which he enters upon his
only in the Rajya Sabha and not in the Lok Sabha. office. However the governor has no security of
However, no such resolution can be moved tenure. He may be removed by the President at
unless at least 14 days’ advance notice has been any time.
given.
· Statement 3 is incorrect: The Vice President  Statement 2 is correct
can be removed from the office before
completion of his term. A formal impeachment is The members of state information commission
not required for his removal. He can be removed are appointed by the Governor on the
by a resolution passed by a majority of all the recommendation of a committee consisting of
then members of the Rajya Sabha and agreed to the Chief Minister as Chairperson, the Leader of
by the Lok Sabha. This means that this resolution Opposition in the Legislative Assembly and a
should be passed in the Rajya Sabha by an State Cabinet Minister nominated by the Chief
effective majority and in the Lok Sabha by a Minister, Governor can also remove the State
simple majority. Chief Information Commissioner or any State
Information Commissioner from the office.
Q.93) Ans: c
Exp:  Statement 3 is incorrect

 Statement 1 is incorrect: The office of Governor cannot pardon a death sentence. Even
governor of the Indian Constitution has been if a state law prescribes a death sentence, the
taken from Government of India Act, 1935.The power to grant pardon lies with the President and
manner of appointment of governor was debated not the governor. However, the governor can
in constituent assembly. The Draft Constitution suspend, remit or commute a death sentence
provided for the direct election of the governor
on the basis of universal adult suffrage. But the Q.95) Ans: b
Constituent Assembly opted for the present Exp:
system of appointment of governor to avoid
conflicts between chief Minister and Governor.  Statement 1 is incorrect: Originally the
Thus the Canadian model, where the governor of constitution prescribed for the Chief justice of
e

a state(province) is appointed by the Centre India and seven judges in Supreme Court and also
in

(Governor-General) is adopted, empowered Parliament of India to increase the


nl

 Statement 2 is correct: The Constitution lays number of judges of the Supreme. However, the
l.o

down that the governor is entitled to such number of judges of the High Court is decided by
ria

emoluments, allowances and privileges as may the President.


be determined by Parliament from time to time.
e

 Statement 2 is correct: A person to be appointed


at

 Statement 3 is incorrect: When the same person


as a judge of the Supreme Court should have the
cm

is appointed as the governor of two or more


states, the emoluments and allowances payable following qualifications:
ps

to him are shared by the states in such


proportion as determined by the president.
.u
w

PRAYAS TEST 4 42
w
w
Contact us : info@onlyias.com

OnlyIAS Nothing Else Visit : dpp.onlyias.in


Contact : +91-7007 931 912

1. He should be a citizen of India. Q.98) Ans: b


2. (a) He should have been a judge of a High Exp:
Court (or high courts in succession) for five years
(b) He should have been an advocate of a High  Statement 1 is correct: The above disqualification
Court (or High Courts in succession) for ten years on the ground of defection does not apply in the
(c) He should be a distinguished jurist in the following two cases:
opinion of the president.  (a) If a member goes out of his party as a result
Thus it is clear that the Constitution has not of a merger of the party with another party. A
prescribed a minimum age for appointment as a merger takes place when two-thirds of the
judge of the Supreme Court. members of the party have agreed to such
merger.
Q.96) Ans: b (b) If a member, after being elected as the
Exp: presiding officer of the House, voluntarily gives
up the membership of his party or rejoins it after
 Statement 1 is incorrect: The constitution does he ceases to hold that office. This exemption has
not provide for minimum or maximum number of been provided in view of the dignity and
judges in a high court. The number of judges of impartiality of this office.
the High Court is decided by the President of  Statement 2 is correct: Any question regarding
India. disqualification arising out of defection is to
 Statement 2 is incorrect: The Parliament can be decided by the presiding officer of the House
extend the jurisdiction of a high court to any
union territory or exclude the jurisdiction of a
high court from any union territory
 Statement 3 is correct: According to  Statement 3 is incorrect: Originally, the act
Representations of People Act, 1951 the disputes provided that the decision of the presiding officer
regarding election of Member of Parliament and is final and cannot be questioned in any court.
member of state legislature is heard by the High However, in the Kihoto Hollohan case , the
Court. This power of High Court falls under its Supreme Court declared this provision as
original jurisdiction. unconstitutional on the ground that it seeks to
take away the jurisdiction of the Supreme Court
Q.97) Ans: c and the high courts. It held that the presiding
Exp: officer, while deciding a question under the Tenth
Schedule, function as a tribunal. Hence, his
 Option 1 is correct: Matters related to revenue decision like that of any other tribunal, is subject
collection falls under the original jurisdiction of to judicial review
High Court
e

 Option 2 is correct: enforcement of fundamental Q.99) Ans: c


in

rights falls under original jurisdiction of High Exp:


nl

Court
l.o

 Option 3 is correct Cases requiring interpretation  Statement 1 is incorrect: The salary and
ria

of constitution that are transferred from allowances of the Chief Minister are determined
subordinate Court falls under original jurisdiction by the state legislature. In addition to the salary
e

and allowances, which are payable to a member


at

of High Court
 Option 4 is incorrect: Dispute relating to election of the state legislature, he gets a sumptuary
cm

of Panchayat members is adjudicated as allowance, free accommodation, travelling


ps

mentioned in state legislative acts. Thus they do allowance, medical facilities


not fall under original jurisdiction of High Court
.u
w

PRAYAS TEST 4 43
w
w
Contact us : info@onlyias.com

OnlyIAS Nothing Else Visit : dpp.onlyias.in


Contact : +91-7007 931 912

 Statement 2 is incorrect: The term of the Chief


Minister is not fixed and he holds office during
the pleasure of the governor. However, this does
not mean that the governor can dismiss him at
any time. He cannot be dismissed by the
governor as long as he enjoys the majority
support in the legislative assembly

Q.100) Ans: d

Exp:

 Statement 1 is correct: The Cabinet Secretary is


ex-officio head of the Civil Services Board and the
Cabinet Secretariat. He is also the head of the IAS
and all civil services under the rules of business of
the Government of India.
 Statement 2 is correct: The Cabinet Secretary is
responsible to provide Secretarial assistance to
the Cabinet and its Committees. He is also
instrumental in attaining inter-ministerial
coordination and smooth functioning among
various Ministries.
 Statement 3 is correct: Cabinet Secretary is also
responsible to evolve consensus through the
instrumentality of a standing as well as ad hoc
committees of the secretaries

e
in
nl
l.o
e ria
at
cm
ps
.u
w

PRAYAS TEST 4 44
w
w

You might also like